NG304- Practice question Exam 1 - Chapter 8,9,10,11,2,3,4,5,6,7

Ace your homework & exams now with Quizwiz!

19. Which technique is correct when the nurse is assessing the radial pulse of a patient? The pulse is counted for: a. 1 minute, if the rhythm is irregular. b. 15 seconds and then multiplied by 4, if the rhythm is regular. c. 2 full minutes to detect any variation in amplitude. d. 10 seconds and then multiplied by 6, if the patient has no history of cardiac abnormalities.

a. 1 minute, if the rhythm is irregular. Recent research suggests that the 30-second interval multiplied by 2 is the most accurate and efficient technique when heart rates are normal or rapid and when rhythms are regular. If the rhythm is irregular, then the pulse is counted for 1 full minute.

33. When auscultating the blood pressure of a 25-year-old patient, the nurse notices the phase I Korotkoff sounds begin at 200 mm Hg. At 100 mm Hg, the Korotkoff sounds muffle. At 92 mm Hg, the Korotkoff sounds disappear. How should the nurse record this patients blood pressure? a. 200/92 b. 200/100 c. 100/200/92 d. 200/100/92

a. 200/92 In adults, the last audible sound best indicates the diastolic pressure. When a variance is greater than 10 to 12 mm Hg between phases IV and V, both phases should be recorded along with the systolic reading (e.g., 142/98/80).

1. A patient has been unable to eat solid food for 2 weeks and is in the clinic today complaining of weakness, tiredness, and hair loss. The patient states that her usual weight is 175 pounds, but today she weighs 161 pounds. What is her recent weight change percentage? To calculate recent weight change percentage, use this formula: Usual weight current weight 100 usual weight

8% 175 161 = 14 pounds 14 175 = 0.08 0.08 100 = 8% DIF: Cognitive Level: Analyzing (Analysis) REF: p. 189 MSC: Client Needs: Health Promotion and Maintenance

1. While measuring a patients blood pressure, the nurse uses the proper technique to obtain an accurate reading. Which of these situations will result in a falsely high blood pressure reading? Select all that apply. a. The person supports his or her own arm during the blood pressure reading. b. The blood pressure cuff is too narrow for the extremity. c. The arm is held above level of the heart. d. The cuff is loosely wrapped around the arm. e. The person is sitting with his or her legs crossed. f. The nurse does not inflate the cuff high enough.

A, B, D, E a. The person supports his or her own arm during the blood pressure reading. b. The blood pressure cuff is too narrow for the extremity. d. The cuff is loosely wrapped around the arm. e. The person is sitting with his or her legs crossed. Several factors can result in blood pressure readings that are too high or too low. Having the patients arm held above the level of the heart is one part of the correct technique. (Refer to Table 9-5, Common Errors in Blood Pressure Measurement.)

2. A patient visits the clinic to ask about smoking cessation. He has smoked heavily for 30 years and wants to stop cold turkey. He asks the nurse, What symptoms can I expect if I do this? Which of these symptoms should the nurse share with the patient as possible symptoms of nicotine withdrawal? Select all that apply. a. Headaches b. Hunger c. Sleepiness d. Restlessness e. Nervousness f. Sweating

A, B, D, E a. Headaches b. Hunger d. Restlessness e. Nervousness Symptoms of nicotine withdrawal include vasodilation, headaches, anger, irritability, frustration, anxiety, nervousness, awakening at night, difficulty concentrating, depression, hunger, impatience, and the desire to smoke (see Table 6-7).

1. The nurse is assessing a patient who is admitted with possible delirium. Which of these are manifestations of delirium? Select all that apply. a. Develops over a short period. b. Person is experiencing apraxia. c. Person is exhibiting memory impairment or deficits. d. Occurs as a result of a medical condition, such as systemic infection. e. Person is experiencing agnosia.

A, C, D a. Develops over a short period. c. Person is exhibiting memory impairment or deficits. d. Occurs as a result of a medical condition, such as systemic infection. Delirium is a disturbance of consciousness that develops over a short period and may be attributable to a medical condition. Memory deficits may also occur. Apraxia and agnosia occur with dementia.

1. The nurse assesses an older woman and suspects physical abuse. Which questions are appropriate for screening for abuse? Select all that apply. a. Has anyone made you afraid, touched you in ways that you did not want, or hurt you physically? b. Are you being abused? c. Have you relied on people for any of the following: bathing, dressing, shopping, banking, or meals? d. Have you been upset because someone talked to you in a way that made you feel shamed or threatened? e. Have you relied on people for any of the following: bathing, dressing, shopping, banking, or meals?

A, C, D, E a. Has anyone made you afraid, touched you in ways that you did not want, or hurt you physically? c. Have you relied on people for any of the following: bathing, dressing, shopping, banking, or meals? d. Have you been upset because someone talked to you in a way that made you feel shamed or threatened? e. Have you relied on people for any of the following: bathing, dressing, shopping, banking, or meals? Directly asking Are you being abused? is not an appropriate screening question for abuse because the woman could easily say no, and no further information would be obtained. The other questions are among the questions recommended by the Elder Abuse Suspicion Index (EASI) when screening for elder abuse.

1. The nurse is assessing a patients headache pain. Which questions reflect one or more of the critical characteristics of symptoms that should be assessed? Select all that apply. a. Where is the headache pain? b. Did you have these headaches as a child? c. On a scale of 1 to 10, how bad is the pain? d. How often do the headaches occur? e. What makes the headaches feel better? f. Do you have any family history of headaches?

A, C, D, E a. Where is the headache pain? c. On a scale of 1 to 10, how bad is the pain? d. How often do the headaches occur? e. What makes the headaches feel better? The mnemonic PQRSTU may help the nurse remember to address the critical characteristics that need to be assessed: (1) P: provocative or palliative; (2) Q: quality or quantity; (3) R: region or radiation; (4) S: severity scale; (5) T: timing; and (6) U: understand the patients perception. Asking, Where is the pain? reflects region. Asking the patient to rate the pain on a 1 to 10 scale reflects severity. Asking How often reflects timing. Asking what makes the pain better reflects provocative. The other options reflect health history and family history.

2. The nurse is asking questions about a patients health beliefs. Which questions are appropriate? Select all that apply. a. What is your definition of health? b. Does your family have a history of cancer? c. How do you describe illness? d. What did your mother do to keep you from getting sick? e. Have you ever had any surgeries? f. How do you keep yourself healthy?

A, C, D, F a. What is your definition of health? c. How do you describe illness? d. What did your mother do to keep you from getting sick? f. How do you keep yourself healthy? The questions listed are appropriate questions for an assessment of a patients health beliefs and practices. The questions regarding family history and surgeries are part of the patients physical history, not the patients health beliefs.

2. The nurse is conducting an interview in an outpatient clinic and is using a computer to record data. Which are the best uses of the computer in this situation? Select all that apply. a. Collect the patients data in a direct, face-to-face manner. b. Enter all the data as the patient states them. c. Ask the patient to wait as the nurse enters the data. d. Type the data into the computer after the narrative is fully explored. e. Allow the patient to see the monitor during typing.

A, D, E a. Collect the patients data in a direct, face-to-face manner. d. Type the data into the computer after the narrative is fully explored. e. Allow the patient to see the monitor during typing. The use of a computer can become a barrier. The nurse should begin the interview as usual by greeting the patient, establishing rapport, and collecting the patients narrative story in a direct, face-to-face manner. Only after the narrative is fully explored should the nurse type data into the computer. When typing, the nurse should position the monitor so that the patient can see it.

1. During assessment of a patients pain, the nurse is aware that certain nonverbal behaviors are associated with chronic pain. Which of these behaviors are associated with chronic pain? Select all that apply. a. Sleeping b. Moaning c. Diaphoresis d. Bracing e. Restlessness f. Rubbing

A, D, F a. Sleeping d. Bracing f. Rubbing Behaviors that have been associated with chronic pain include bracing, rubbing, diminished activity, sighing, and changes in appetite. In addition, those with chronic pain may sleep in an attempt at distraction. The other behaviors are associated with acute pain.

1. The nurse is assessing a patient who is obese for signs of metabolic syndrome. This condition is diagnosed when three or more certain risk factors are present. Which of these assessment findings are risk factors for metabolic syndrome? Select all that apply. a. Fasting plasma glucose level less than 100 mg/dL b. Fasting plasma glucose level greater than or equal to 110 mg/dL c. Blood pressure reading of 140/90 mm Hg d. Blood pressure reading of 110/80 mm Hg e. Triglyceride level of 120 mg/dL

B, C b. Fasting plasma glucose level greater than or equal to 110 mg/dL c. Blood pressure reading of 140/90 mm Hg Metabolic syndrome is diagnosed when three or more of the following risk factors are present: (1) fasting plasma glucose level greater than or equal to 100 mg/dL; (2) blood pressure greater than or equal to 130/85 mm Hg; (3) waist circumference greater than or equal to 40 inches for men and 35 inches for women; (4) highdensity lipoprotein cholesterol less than 40 in men and less than 50 in women; and (5) triglyceride levels greater than or equal to 150 mg/dL (ATP III, 2001).

2. During an admission assessment of a patient with dementia, the nurse assesses for pain because the patient has recently had several falls. Which of these are appropriate for the nurse to assess in a patient with dementia? Select all that apply. a. Ask the patient, Do you have pain? b. Assess the patients breathing independent of vocalization. c. Note whether the patient is calling out, groaning, or crying. d. Have the patient rate pain on a 1-to-10 scale. e. Observe the patients body language for pacing and agitation.

B, C, E b. Assess the patients breathing independent of vocalization. c. Note whether the patient is calling out, groaning, or crying. e. Observe the patients body language for pacing and agitation. Patients with dementia may say no when, in reality, they are very uncomfortable because words have lost their meaning. Patients with dementia become less able to identify and describe pain over time, although pain is still present. People with dementia communicate pain through their behaviors. Agitation, pacing, and repetitive yelling may indicate pain and not a worsening of the dementia. (See Figure 10-10 for the Pain Assessment in Advanced Dementia [PAINAD] scale, which may also be used to assess pain in persons with dementia.)

1. A patient with a known history of heavy alcohol use has been admitted to the ICU after he was found unconscious outside a bar. The nurse closely monitors him for symptoms of withdrawal. Which of these symptoms may occur during this time? Select all that apply. a. Bradycardia b. Coarse tremor of the hands c. Transient hallucinations d. Somnolence e. Sweating

B, C, E b. Coarse tremor of the hands c. Transient hallucinations e. Sweating Symptoms of uncomplicated alcohol withdrawal start shortly after the cessation of drinking, peak at the second day, and improve by the fourth or fifth day. Symptoms include coarse tremors of the hands, tongue, and eyelids; anorexia; nausea and vomiting; autonomic hyperactivity (e.g., tachycardia, sweating, elevated blood pressure); and transient hallucinations, among other symptoms (see Table 6-7).

2. The nurse is conducting a developmental history on a 5-year-old child. Which questions are appropriate to ask the parents for this part of the assessment? Select all that apply. a. How much junk food does your child eat? b. How many teeth has he lost, and when did he lose them? c. Is he able to tie his shoelaces? d. Does he take a childrens vitamin? e. Can he tell time? f. Does he have any food allergies?

B, C, E b. How many teeth has he lost, and when did he lose them? c. Is he able to tie his shoelaces? e. Can he tell time? Questions about tooth loss, ability to tell time, and ability to tie shoelaces are appropriate questions for a developmental assessment. Questions about junk food intake and vitamins are part of a nutritional history. Questions about food allergies are not part of a developmental history.

1. The nurse is conducting an interview. Which of these statements is true regarding open-ended questions? Select all that apply. a. Open-ended questions elicit cold facts. b. They allow for self-expression. c. Open-ended questions build and enhance rapport. d. They leave interactions neutral. e. Open-ended questions call for short one- to two-word answers. f. They are used when narrative information is needed.

B, C, F b. They allow for self-expression. c. Open-ended questions build and enhance rapport. f. They are used when narrative information is needed. Open-ended questions allow for self-expression, build and enhance rapport, and obtain narrative information. These features enhance communication during an interview. The other statements are appropriate for closed or direct questions.

1. The nurse is reviewing aspects of cultural care. Which statements illustrate proper cultural care? Select all that apply. a. Examine the patient within the context of ones own cultural health and illness practices. b. Select questions that are not complex. c. Ask questions rapidly. d. Touch patients within the cultural boundaries of their heritage. e. Pace questions throughout the physical examination.

B, D, E b. Select questions that are not complex. d. Touch patients within the cultural boundaries of their heritage. e. Pace questions throughout the physical examination. Patients should be examined within the context of their own cultural health and illness practices. Questions should be simply stated and not rapidly asked.

1. What is the pulse pressure for a patient whose blood pressure is 158/96 mm Hg and whose pulse rate is 72 beats per minute?

The pulse pressure is the difference between the systolic and diastolic and reflects the stroke volume. The pulse rate is not necessary for pulse pressure calculations.

19. The nurse is evaluating patients for obesity-related diseases by calculating the waist-to-hip ratios. Which one of these patients would be at increased risk? a. 29-year-old woman whose waist measures 33 inches and hips measure 36 inches b. 32-year-old man whose waist measures 34 inches and hips measure 36 inches c. 38-year-old man whose waist measures 35 inches and hips measure 38 inches d. 46-year-old woman whose waist measures 30 inches and hips measure 38 inches

a. 29-year-old woman whose waist measures 33 inches and hips measure 36 inches The waist-to-hip ratio assesses body fat distribution as an indicator of health risk. A waist-to-hip ratio of 1.0 or greater in men or 0.8 or greater in women is indicative of android (upper body obesity) and increasing risk for obesity-related disease and early death. The 29-year-old woman has a waist-to-hip ratio of 0.92, which is greater than 0.8. The 32-year-old man has a waist-to-hip ratio of 0.94; the 38-year-old man has a waist-to-hip ratio of 0.92; the 46-year-old woman has a waist-to-hip ratio of 0.78.

The nurse recognizes which of these persons is at greatest risk for undernutrition? a. 5-month-old infant b. 50-year-old woman c. 20-year-old college student d. 30-year-old hospital administrator

a. 5-month-old infant Vulnerable groups for undernutrition are infants, children, pregnant women, recent immigrants, persons with low incomes, hospitalized people, and aging adults.

9. The nurse is providing care for a 68-year-old woman who is complaining of constipation. What concern exists regarding her nutritional status? a. Absorption of nutrients may be impaired. b. Constipation may represent a food allergy. c. The patient may need emergency surgery to correct the problem. d. Gastrointestinal problems will increase her caloric demand

a. Absorption of nutrients may be impaired. Gastrointestinal symptoms such as vomiting, diarrhea, or constipation may interfere with nutrient intake or absorption. The other responses are not correct.

2. The nurse is assessing the mental status of a child. Which statement about children and mental status is true? a. All aspects of mental status in children are interdependent. b. Children are highly labile and unstable until the age of 2 years. c. Childrens mental status is largely a function of their parents level of functioning until the age of 7 years. d. A childs mental status is impossible to assess until the child develops the ability to concentrate.

a. All aspects of mental status in children are interdependent. Separating and tracing the development of only one aspect of mental status is difficult. All aspects are interdependent. For example, consciousness is rudimentary at birth because the cerebral cortex is not yet developed. The infant cannot distinguish the self from the mothers body. The other statements are not true.

29. A student is late for his appointment and has rushed across campus to the health clinic. The nurse should: a. Allow 5 minutes for him to relax and rest before checking his vital signs. b. Check the blood pressure in both arms, expecting a difference in the readings because of his recent exercise. c. Immediately monitor his vital signs on his arrival at the clinic and then 5 minutes later, recording any differences. d. Check his blood pressure in the supine position, which will provide a more accurate reading and will allow him to relax at the same time.

a. Allow 5 minutes for him to relax and rest before checking his vital signs. A comfortable, relaxed person yields a valid blood pressure. Many people are anxious at the beginning of an examination; the nurse should allow at least a 5-minute rest period before measuring blood pressure.

25. A 75-year-old woman is at the office for a preoperative interview. The nurse is aware that the interview may take longer than interviews with younger persons. What is the reason for this? a. An aged person has a longer story to tell. b. An aged person is usually lonely and likes to have someone with whom to talk. c. Aged persons lose much of their mental abilities and require longer time to complete an interview. d. As a person ages, he or she is unable to hear; thus the interviewer usually needs to repeat much of what is said.

a. An aged person has a longer story to tell. The interview usually takes longer with older adults because they have a longer story to tell. It is not necessarily true that all older adults are lonely, have lost mental abilities, or are hard of hearing.

21. A man is at the clinic for a physical examination. He states that he is very anxious about the physical examination. What steps can the nurse take to make him more comfortable? a. Appear unhurried and confident when examining him. b. Stay in the room when he undresses in case he needs assistance. c. Ask him to change into an examining gown and to take off his undergarments. d. Defer measuring vital signs until the end of the examination, which allows him time to become comfortable.

a. Appear unhurried and confident when examining him. Anxiety can be reduced by an examiner who is confident, self-assured, considerate, and unhurried. Familiar and relatively nonthreatening actions, such as measuring the persons vital signs, will gradually accustom the person to the examination.

6. During an assessment, the nurse notices that a patient is handling a small charm that is tied to a leather strip around his neck. Which action by the nurse is appropriate? a. Ask the patient about the item and its significance. b. Ask the patient to lock the item with other valuables in the hospitals safe. c. Tell the patient that a family member should take valuables home. d. No action is necessary.

a. Ask the patient about the item and its significance. The nurse should inquire about the amulets meaning. Amulets, such as charms, are often considered an important means of protection from evil spirits by some cultures.

11. A woman who has lived in the United States for a year after moving from Europe has learned to speak English and is almost finished with her college studies. She now dresses like her peers and says that her family in Europe would hardly recognize her. This nurse recognizes that this situation illustrates which concept? a. Assimilation b. Heritage consistency c. Biculturalism d. Acculturation

a. Assimilation Assimilation is the process by which a person develops a new cultural identity and becomes like members of the dominant culture. This concept does not reflect heritage consistency. Biculturalism is a dual pattern of identification; acculturation is the process of adapting to and acquiring another culture.

18. In teaching a patient how to determine total body fat at home, the nurse includes instructions to obtain measurements of: a. Height and weight. b. Frame size and weight. c. Waist and hip circumferences. d. Mid-upper arm circumference and arm span.

a. Height and weight. Body mass index, calculated by using height and weight measurements, is a practical marker of optimal weight for height and an indicator of obesity. The other options are not correct.

28. A 6-month-old infant has been brought to the well-child clinic for a check-up. She is currently sleeping. What should the nurse do first when beginning the examination? a. Auscultate the lungs and heart while the infant is still sleeping. b. Examine the infants hips, because this procedure is uncomfortable. c. Begin with the assessment of the eye, and continue with the remainder of the examination in a head-to-toe approach. d. Wake the infant before beginning any portion of the examination to obtain the most accurate assessment of body systems.

a. Auscultate the lungs and heart while the infant is still sleeping. When the infant is quiet or sleeping is an ideal time to assess the cardiac, respiratory, and abdominal systems. Assessment of the eye, ear, nose, and throat are invasive procedures that should be performed at the end of the examination.

10. When taking a history from a newly admitted patient, the nurse notices that he often pauses and expectantly looks at the nurse. What would be the nurses best response to this behavior? a. Be silent, and allow him to continue when he is ready. b. Smile at him and say, Dont worry about all of this. Im sure we can find out why youre having these pains. c. Lean back in the chair and ask, You are looking at me kind of funny; there isnt anything wrong, is there? d. Stand up and say, I can see that this interview is uncomfortable for you. We can continue it another time.

a. Be silent, and allow him to continue when he is ready. Silent attentiveness communicates that the person has time to think and to organize what he or she wishes to say without an interruption from the nurse. Health professionals most often interrupt this thinking silence. The other responses are not conducive to ideal communication.

45. A 75-year-old man with a history of hypertension was recently changed to a new antihypertensive drug. He reports feeling dizzy at times. How should the nurse evaluate his blood pressure? a. Blood pressure and pulse should be recorded in the supine, sitting, and standing positions. b. The patient should be directed to walk around the room and his blood pressure assessed after this activity. c. Blood pressure and pulse are assessed at the beginning and at the end of the examination. d. Blood pressure is taken on the right arm and then 5 minutes later on the left arm.

a. Blood pressure and pulse should be recorded in the supine, sitting, and standing positions. Orthostatic vital signs should be taken when the person is hypertensive or is taking antihypertensive medications, when the person reports fainting or syncope, or when volume depletion is suspected. The blood pressure and pulse readings are recorded in the supine, sitting, and standing positions.

4. A patient tells the nurse that she has had abdominal pain for the past week. What would be the nurses best response? a. Can you point to where it hurts? b. Well talk more about that later in the interview. c. What have you had to eat in the last 24 hours? d. Have you ever had any surgeries on your abdomen?

a. Can you point to where it hurts? A final summary of any symptom the person has should include, along with seven other critical characteristics, Location: specific. The person is asked to point to the location.

10. During a nutritional assessment, why is it important for the nurse to ask a patient what medications he or she is taking? a. Certain drugs can affect the metabolism of nutrients. b. The nurse needs to assess the patient for allergic reactions. c. Medications need to be documented in the record for the physicians review. d. Medications can affect ones memory and ability to identify food eaten in the last 24 hours

a. Certain drugs can affect the metabolism of nutrients. Analgesics, antacids, anticonvulsants, antibiotics, diuretics, laxatives, antineoplastic drugs, steroids, and oral contraceptives are drugs that can interact with nutrients, impairing their digestion, absorption, metabolism, or use. The other responses are not correct.

27. The nurse is asking a patient for his reason for seeking care and asks about the signs and symptoms he is experiencing. Which of these is an example of a symptom? a. Chest pain b. Clammy skin c. Serum potassium level at 4.2 mEq/L d. Body temperature of 100 F

a. Chest pain A symptom is a subjective sensation (e.g., chest pain) that a person feels from a disorder. A sign is an objective abnormality that the examiner can detect on physical examination or in laboratory reports, as illustrated by the other responses.

25. The nurse recognizes that working with children with a different cultural perspective may be especially difficult because: a. Children have spiritual needs that are influenced by their stages of development. b. Children have spiritual needs that are direct reflections of what is occurring in their homes. c. Religious beliefs rarely affect the parents perceptions of the illness. d. Parents are often the decision makers, and they have no knowledge of their childrens spiritual needs.

a. Children have spiritual needs that are influenced by their stages of development. Illness during childhood may be an especially difficult clinical situation. Children, as well as adults, have spiritual needs that vary according to the childs developmental level and the religious climate that exists in the family. The other statements are not correct.

37. The nurse is administering a Mini-Cog test to an older adult woman. When asked to draw a clock showing the time of 10:45, the patient drew a clock with the numbers out of order and with an incorrect time. This result indicates which finding? a. Cognitive impairment b. Amnesia c. Delirium d. Attention-deficit disorder

a. Cognitive impairment The Mini-Cog is a newer instrument that screens for cognitive impairment, often found with dementia. The result of an abnormal drawing of a clock and time indicates a cognitive impairment.

9. When percussing over the liver of a patient, the nurse notices a dull sound. The nurse should: a. Consider this a normal finding. b. Palpate this area for an underlying mass. c. Reposition the hands, and attempt to percuss in this area again. d. Consider this finding as abnormal, and refer the patient for additional treatment.

a. Consider this a normal finding. Percussion over relatively dense organs, such as the liver or spleen, will produce a dull sound. The other responses are not correct.

26. A 30-year-old woman has recently moved to the United States with her husband. They are living with the womans sister until they can get a home of their own. When company arrives to visit with the womans sister, the woman feels suddenly shy and retreats to the back bedroom to hide until the company leaves. She explains that her reaction to guests is simply because she does not know how to speak perfect English. This woman could be experiencing: a. Culture shock. b. Cultural taboos. c. Cultural unfamiliarity. d. Culture disorientation.

a. Culture shock. Culture shock is a term used to describe the state of disorientation or inability to respond to the behavior of a different cultural group because of its sudden strangeness, unfamiliarity, and incompatibility with the individuals perceptions and expectations. The other terms are not correct.

26. The nurse is interviewing a male patient who has a hearing impairment. What techniques would be most beneficial in communicating with this patient? a. Determine the communication method he prefers. b. Avoid using facial and hand gestures because most hearing-impaired people find this degrading. c. Request a sign language interpreter before meeting with him to help facilitate the communication. d. Speak loudly and with exaggerated facial movement when talking with him because doing so will help him lip read.

a. Determine the communication method he prefers. The nurse should ask the deaf person the preferred way to communicateby signing, lip reading, or writing. If the person prefers lip reading, then the nurse should be sure to face him squarely and have good lighting on the nurses face. The nurse should not exaggerate lip movements because this distorts words. Similarly, shouting distorts the reception of a hearing aid the person may wear. The nurse should speak slowly and supplement his or her voice with appropriate hand gestures or pantomime.

1. A woman has come to the clinic to seek help with a substance abuse problem. She admits to using cocaine just before arriving. Which of these assessment findings would the nurse expect to find when examining this woman? a. Dilated pupils, pacing, and psychomotor agitation b. Dilated pupils, unsteady gait, and aggressiveness c. Pupil constriction, lethargy, apathy, and dysphoria d. Constricted pupils, euphoria, and decreased temperature

a. Dilated pupils, pacing, and psychomotor agitation A cocaine users appearance includes pupillary dilation, tachycardia or bradycardia, elevated or lowered blood pressure, sweating, chills, nausea, vomiting, and weight loss. The persons behavior includes euphoria, talkativeness, hypervigilance, pacing, psychomotor agitation, impaired social or occupational functioning, fighting, grandiosity, and visual or tactile hallucinations.

12. The nurse is obtaining a history from a 30-year-old male patient and is concerned about health promotion activities. Which of these questions would be appropriate to use to assess health promotion activities for this patient? a. Do you perform testicular self-examinations? b. Have you ever noticed any pain in your testicles? c. Have you had any problems with passing urine? d. Do you have any history of sexually transmitted diseases?

a. Do you perform testicular self-examinations? Health promotion for a man would include the performance of testicular self-examinations. The other questions are asking about possible disease or illness issues.

30. The nurse is performing a health interview on a patient who has a language barrier, and no interpreter is available. Which is the best example of an appropriate question for the nurse to ask in this situation? a. Do you take medicine? b. Do you sterilize the bottles? c. Do you have nausea and vomiting? d. You have been taking your medicine, havent you?

a. Do you take medicine? In a situation during which a language barrier exists and no interpreter is available, simple words should be used, avoiding medical jargon. The use of contractions and pronouns should also be avoided. Nouns should be repeatedly used, and one topic at a time should be discussed.

38. During morning rounds, the nurse asks a patient, How are you today? The patient responds, You today, you today, you today! and mumbles the words. This speech pattern is an example of: a. Echolalia b. Clanging c. Word salad d. Perseveration

a. Echolalia Echolalia occurs when a person imitates or repeats anothers words or phrases, often with a mumbling, mocking, or a mechanical tone.

15. An Asian-American woman is experiencing diarrhea, which is believed to be cold or yin. The nurse expects that the woman is likely to try to treat it with: a. Foods that are hot or yang. b. Readings and Eastern medicine meditations. c. High doses of medicines believed to be cold. d. No treatment is tried because diarrhea is an expected part of life.

a. Foods that are hot or yang. Yin foods are cold and yang foods are hot. Cold foods are eaten with a hot illness, and hot foods are eaten with a cold illness. The other explanations do not reflect the yin/yang theory.

20. When assessing a patients pulse, the nurse should also notice which of these characteristics? a. Force b. Pallor c. Capillary refill time d. Timing in the cardiac cycle

a. Force The pulse is assessed for rate, rhythm, and force.

4. During an examination of a child, the nurse considers that physical growth is the best index of a childs: a. General health. b. Genetic makeup. c. Nutritional status. d. Activity and exercise patterns.

a. General health. Physical growth is the best index of a childs general health; recording the childs height and weight helps determine normal growth patterns.

34. The nurse discovers speech problems in a patient during an assessment. The patient has spontaneous speech, but it is mostly absent or is reduced to a few stereotypical words or sounds. This finding reflects which type of aphasia? a. Global b. Brocas c. Dysphonic d. Wernickes

a. Global Global aphasia is the most common and severe form of aphasia. Spontaneous speech is absent or reduced to a few stereotyped words or sounds, and prognosis for language recovery is poor. (Brocas aphasia and Wernickes aphasia are described in Table 5-4.) Dysphonic aphasia is not a valid condition.

23. A patient describes feeling an unreasonable, irrational fear of snakes. His fear is so persistent that he can no longer comfortably look at even pictures of snakes and has made an effort to identify all the places he might encounter a snake and avoids them. The nurse recognizes that he: a. Has a snake phobia. b. Is a hypochondriac; snakes are usually harmless. c. Has an obsession with snakes. d. Has a delusion that snakes are harmful, which must stem from an early traumatic incident involving snakes.

a. Has a snake phobia. A phobia is a strong, persistent, irrational fear of an object or situation; the person feels driven to avoid it. (See Table 5-7 for the definitions of the other terms.)

27. When preparing to perform a physical examination on an infant, the nurse should: a. Have the parent remove all clothing except the diaper on a boy. b. Instruct the parent to feed the infant immediately before the examination. c. Encourage the infant to suck on a pacifier during the abdominal examination. d. Ask the parent to leave the room briefly when assessing the infants vital signs

a. Have the parent remove all clothing except the diaper on a boy. The parent should always be present to increase the childs feeling of security and to understand normal growth and development. The timing of the examination should be 1 to 2 hours after feeding when the baby is neither too drowsy nor too hungry. Infants do not object to being nude; clothing should be removed, but a diaper should be left on a boy.

15. When considering a nutritional assessment, the nurse is aware that the most common anthropometric measurements include: a. Height and weight. b. Leg circumference. c. Skinfold thickness of the biceps. d. Hip and waist measurements.

a. Height and weight. The most commonly used anthropometric measures are height, weight, triceps skinfold thickness, elbow breadth, and arm and head circumferences

5. When reviewing the demographics of ethnic groups in the United States, the nurse recalls that the largest and fastest growing population is: a. Hispanic. b. Black. c. Asian. d. American Indian.

a. Hispanic. Hispanics are the largest and fastest growing population in the United States, followed by Asians, Blacks, American Indians and Alaska natives, and other groups.

11. During a mental status examination, the nurse wants to assess a patients affect. The nurse should ask the patient which question? a. How do you feel today? b. Would you please repeat the following words? c. Have these medications had any effect on your pain? d. Has this pain affected your ability to get dressed by yourself?

a. How do you feel today? Judge mood and affect by body language and facial expression and by directly asking, How do you feel today? or How do you usually feel? The mood should be appropriate to the persons place and condition and should appropriately change with the topics.

12. The nurse is performing a nutritional assessment on a 15-year-old girl who tells the nurse that she is so fat. Assessment reveals that she is 5 feet 4 inches and weighs 110 pounds. The nurses appropriate response would be: a. How much do you think you should weigh? b. Dont worry about it; youre not that overweight. c. The best thing for you would be to go on a diet. d. I used to always think I was fat when I was your age.

a. How much do you think you should weigh? Adolescents increased body awareness and self-consciousness may cause eating disorders such as anorexia nervosa or bulimia, conditions in which the real or perceived body image does not favorably compare with an ideal image. The nurse should not belittle the adolescents feelings, provide unsolicited advice, or agree with her.

2. The nurse is assessing a patient who has been admitted for cirrhosis of the liver, secondary to chronic alcohol use. During the physical assessment, the nurse looks for cardiac problems that are associated with chronic use of alcohol, such as: a. Hypertension. b. Ventricular fibrillation. c. Bradycardia. d. Mitral valve prolapse.

a. Hypertension. Even moderate drinking leads to hypertension and cardiomyopathy, with an increase in left ventricular mass, dilation of ventricles, and wall thinning. Ventricular fibrillation, bradycardia, and mitral valve prolapse are not associated with chronic heavy use of alcohol.

47. The nurse is assessing an 8-year-old child whose growth rate measures below the third percentile for a child his age. He appears significantly younger than his stated age and is chubby with infantile facial features. Which condition does this child have? a. Hypopituitary dwarfism b. Achondroplastic dwarfism c. Marfan syndrome d. Acromegaly

a. Hypopituitary dwarfism Hypopituitary dwarfism is caused by a deficiency in growth hormone in childhood and results in a retardation of growth below the third percentile, delayed puberty, and other problems. The childs appearance fits this description. Achondroplastic dwarfism is a genetic disorder resulting in characteristic deformities; Marfan syndrome is an inherited connective tissue disorder characterized by a tall, thin stature and other features. Acromegaly is the result of excessive secretion of growth hormone in adulthood. (For more information, see Table 9-5, Abnormalities in Body Height and Proportion.)

14. The nurse is preparing to use a stethoscope for auscultation. Which statement is true regarding the diaphragm of the stethoscope? The diaphragm: a. Is used to listen for high-pitched sounds. b. Is used to listen for low-pitched sounds. c. Should be lightly held against the persons skin to block out low-pitched sounds. d. Should be lightly held against the persons skin to listen for extra heart sounds and murmurs.

a. Is used to listen for high-pitched sounds. The diaphragm of the stethoscope is best for listening to high-pitched sounds such as breath, bowel, and normal heart sounds. It should be firmly held against the persons skin, firmly enough to leave a ring. The bell of the stethoscope is best for soft, low-pitched sounds such as extra heart sounds or murmurs.

19. A patient drifts off to sleep when she is not being stimulated. The nurse can easily arouse her by calling her name, but the patient remains drowsy during the conversation. The best description of this patients level of consciousness would be: a. Lethargic b. Obtunded c. Stuporous d. Semialert

a. Lethargic Lethargic (or somnolent) is when the person is not fully alert, drifts off to sleep when not stimulated, and can be aroused when called by name in a normal voice but looks drowsy. He or she appropriately responds to questions or commands, but thinking seems slow and fuzzy. He or she is inattentive and loses the train of thought. Spontaneous movements are decreased. (See Table 5-3 for the definitions of the other terms.)

3. The nurse is providing nutrition information to the mother of a 1-year-old child. Which of these statements represents accurate information for this age group? a. Maintaining adequate fat and caloric intake is important for a child in this age group. b. The recommended dietary allowances for an infant are the same as for an adolescent. c. The babys growth is minimal at this age; therefore, caloric requirements are decreased. d. The baby should be placed on skim milk to decrease the risk of coronary artery disease when he or she grows older.

a. Maintaining adequate fat and caloric intake is important for a child in this age group. Because of rapid growth, especially of the brain, both infants and children younger than 2 years of age should not drink skim or low-fat milk or be placed on low-fat diets. Fats (calories and essential fatty acids) are required for proper growth and central nervous system development.

25. During reporting, the nurse hears that a patient is experiencing hallucinations. Which is an example of a hallucination? a. Man believes that his dead wife is talking to him. b. Woman hears the doorbell ring and goes to answer it, but no one is there. c. Child sees a man standing in his closet. When the lights are turned on, it is only a dry cleaning bag. d. Man believes that the dog has curled up on the bed, but when he gets closer he sees that it is a blanket.

a. Man believes that his dead wife is talking to him. Hallucinations are sensory perceptions for which no external stimuli exist. They may strike any sense: visual, auditory, tactile, olfactory, or gustatory.

17. Illness is considered part of lifes rhythmic course and is an outward sign of disharmony within. This statement most accurately reflects the views about illness from which theory? a. Naturalistic b. Biomedical c. Reductionist d. Magicoreligious

a. Naturalistic The naturalistic perspective states that the laws of nature create imbalances, chaos, and disease. From the perspective of the Chinese, for example, illness is not considered an introducing agent; rather, illness is considered a part of lifes rhythmic course and an outward sign of disharmony within. The other options are not correct.

4. In an interview, the nurse may find it necessary to take notes to aid his or her memory later. Which statement is true regarding note-taking? a. Note-taking may impede the nurses observation of the patients nonverbal behaviors. b. Note-taking allows the patient to continue at his or her own pace as the nurse records what is said. c. Note-taking allows the nurse to shift attention away from the patient, resulting in an increased comfort level. d. Note-taking allows the nurse to break eye contact with the patient, which may increase his or her level of comfort.

a. Note-taking may impede the nurses observation of the patients nonverbal behaviors. The use of history forms and note-taking may be unavoidable. However, the nurse must be aware that notetaking during the interview has disadvantages. It breaks eye contact too often and shifts the attention away from the patient, which diminishes his or her sense of importance. Note-taking may also interrupt the patients narrative flow, and it impedes the observation of the patients nonverbal behavior.

16. If a 29-year-old woman weighs 156 pounds, and the nurse determines her ideal body weight to be 120 pounds, then how would the nurse classify the womans weight? a. Obese b. Mildly overweight c. Suffering from malnutrition d. Within appropriate range of ideal weight

a. Obese Obesity, as a result of caloric excess, refers to weight more than 20% above ideal body weight. For this patient, 20% of her ideal body weight would be 24 pounds, and greater than 20% of her body weight would be over 144 pounds. Therefore, having a weight of 156 pounds would be considered obese.

1. The nurse is performing a general survey. Which action is a component of the general survey? a. Observing the patients body stature and nutritional status b. Interpreting the subjective information the patient has reported c. Measuring the patients temperature, pulse, respirations, and blood pressure d. Observing specific body systems while performing the physical assessment

a. Observing the patients body stature and nutritional status The general survey is a study of the whole person that includes observing the patients physical appearance, body structure, mobility, and behavior.

16. The nurse will use which technique of assessment to determine the presence of crepitus, swelling, and pulsations? a. Palpation b. Inspection c. Percussion d. Auscultation

a. Palpation Palpation applies the sense of touch to assess texture, temperature, moisture, organ location and size, as well as any swelling, vibration or pulsation, rigidity or spasticity, crepitation, presence of lumps or masses, and the presence of tenderness or pain.

4. Which of these techniques uses the sense of touch to assess texture, temperature, moisture, and swelling when the nurse is assessing a patient? a. Palpation b. Inspection c. Percussion d. Auscultation

a. Palpation Palpation uses the sense of touch to assess the patient for these factors. Inspection involves vision; percussion assesses through the use of palpable vibrations and audible sounds; and auscultation uses the sense of hearing.

11. The nurse is reviewing the principles of nociception. During which phase of nociception does the conscious awareness of a painful sensation occur? a. Perception b. Modulation c. Transduction d. Transmission

a. Perception Perception is the third phase of nociception and indicates the conscious awareness of a painful sensation. During this phase, the sensation is recognized by higher cortical structures and identified as pain.

8. The nurse is reviewing percussion techniques with a newly graduated nurse. Which technique, if used by the new nurse, indicates that more review is needed? a. Percussing once over each area b. Quickly lifting the striking finger after each stroke c. Striking with the fingertip, not the finger pad d. Using the wrist to make the strikes, not the arm

a. Percussing once over each area For percussion, the nurse should percuss two times over each location. The striking finger should be quickly lifted because a resting finger damps off vibrations. The tip of the striking finger should make contact, not the pad of the finger. The wrist must be relaxed and is used to make the strikes, not the arm.

6. A woman brings her husband to the clinic for an examination. She is particularly worried because after a recent fall, he seems to have lost a great deal of his memory of recent events. Which statement reflects the nurses best course of action? a. Perform a complete mental status examination. b. Refer him to a psychometrician. c. Plan to integrate the mental status examination into the history and physical examination. d. Reassure his wife that memory loss after a physical shock is normal and will soon subside.

a. Perform a complete mental status examination. Performing a complete mental status examination is necessary when any abnormality in affect or behavior is discovered or when family members are concerned about a persons behavioral changes (e.g., memory loss, inappropriate social interaction) or after trauma, such as a head injury.

7. When documenting IPV and elder abuse, the nurse should include: a. Photographic documentation of the injuries. b. Summary of the abused patients statements. c. Verbatim documentation of every statement made. d. General description of injuries in the progress notes.

a. Photographic documentation of the injuries. Documentation of IPV and elder abuse must include detailed nonbiased progress notes, the use of injury maps, and photographic documentation. Written documentation needs to be verbatim, within reason. Not every statement can be documented.

8. A patient is admitted to the unit after an automobile accident. The nurse begins the mental status examination and finds that the patient has dysarthric speech and is lethargic. The nurses best approach regarding this examination is to: a. Plan to defer the rest of the mental status examination. b. Skip the language portion of the examination, and proceed onto assessing mood and affect. c. Conduct an in-depth speech evaluation, and defer the mental status examination to another time. d. Proceed with the examination, and assess the patient for suicidal thoughts because dysarthria is often accompanied by severe depression.

a. Plan to defer the rest of the mental status examination. In the mental status examination, the sequence of steps forms a hierarchy in which the most basic functions (consciousness, language) are assessed first. The first steps must be accurately assessed to ensure validity of the steps that follow. For example, if consciousness is clouded, then the person cannot be expected to have full attention and to cooperate with new learning. If language is impaired, then a subsequent assessment of new learning or abstract reasoning (anything that requires language functioning) can give erroneous conclusions.

9. The nurse knows that one advantage of the tympanic membrane thermometer (TMT) is that: a. Rapid measurement is useful for uncooperative younger children. b. Using the TMT is the most accurate method for measuring body temperature in newborn infants. c. Measuring temperature using the TMT is inexpensive. d. Studies strongly support the use of the TMT in children under the age 6 years.

a. Rapid measurement is useful for uncooperative younger children. The TMT is useful for young children who may not cooperate for oral temperatures and fear rectal temperatures. However, the use a TMT with newborn infants and young children is conflicting.

10. The nurse is assessing bruising on an injured patient. Which color indicates a new bruise that is less than 2 hours old? a. Red b. Purple-blue c. Greenish-brown d. Brownish-yellow

a. Red A new bruise is usually red and will often develop a purple or purple-blue appearance 12 to 36 hours after blunt-force trauma. The color of bruises (and ecchymoses) generally progresses from purple-blue to bluishgreen to greenish-brown to brownish-yellow before fading away.

3. The nurse makes which adjustment in the physical environment to promote the success of an interview? a. Reduces noise by turning off televisions and radios b. Reduces the distance between the interviewer and the patient to 2 feet or less c. Provides a dim light that makes the room cozy and helps the patient relax d. Arranges seating across a desk or table to allow the patient some personal space

a. Reduces noise by turning off televisions and radios The nurse should reduce noise by turning off the television, radio, and other unnecessary equipment, because multiple stimuli are confusing. The interviewer and patient should be approximately 4 to 5 feet apart; the room should be well-lit, enabling the interviewer and patient to see each other clearly. Having a table or desk in between the two people creates the idea of a barrier; equal-status seating, at eye level, is better.

37. The nurse is preparing to measure the vital signs of a 6-month-old infant. Which action by the nurse is correct? a. Respirations are measured; then pulse and temperature. b. Vital signs should be measured more frequently than in an adult. c. Procedures are explained to the parent, and the infant is encouraged to handle the equipment. d. The nurse should first perform the physical examination to allow the infant to become more familiar with her and then measure the infants vital signs.

a. Respirations are measured; then pulse and temperature. With an infant, the order of vital sign measurements is reversed to respiration, pulse, and temperature. Taking the temperature first, especially if it is rectal, may cause the infant to cry, which will increase the respiratory and pulse rate, thus masking the normal resting values. The vital signs are measured with the same purpose and frequency as would be measured in an adult.

24. The nurse is assessing the vital signs of a 3-year-old patient who appears to have an irregular respiratory pattern. How should the nurse assess this childs respirations? a. Respirations should be counted for 1 full minute, noticing rate and rhythm. b. Childs pulse and respirations should be simultaneously checked for 30 seconds. c. Childs respirations should be checked for a minimum of 5 minutes to identify any variations in his or her respiratory pattern. d. Patients respirations should be counted for 15 seconds and then multiplied by 4 to obtain the number of respirations per minute.

a. Respirations should be counted for 1 full minute, noticing rate and rhythm. Respirations are counted for 1 full minute if an abnormality is suspected. The other responses are not correct actions.

23. The nurse is performing a nutritional assessment on an 80-year-old patient. The nurse knows that physiologic changes can directly affect the nutritional status of the older adult and include: a. Slowed gastrointestinal motility. b. Hyperstimulation of the salivary glands. c. Increased sensitivity to spicy and aromatic foods. d. Decreased gastrointestinal absorption causing esophageal reflux.

a. Slowed gastrointestinal motility. Normal physiologic changes in aging adults that affect nutritional status include slowed gastrointestinal motility, decreased gastrointestinal absorption, diminished olfactory and taste sensitivity, decreased saliva production, decreased visual acuity, and poor dentition.

13. When evaluating the temperature of older adults, the nurse should remember which aspect about an older adults body temperature? a. The body temperature of the older adult is lower than that of a younger adult. b. An older adults body temperature is approximately the same as that of a young child. c. Body temperature depends on the type of thermometer used. d. In the older adult, the body temperature varies widely because of less effective heat control mechanisms.

a. The body temperature of the older adult is lower than that of a younger adult. In older adults, the body temperature is usually lower than in other age groups, with a mean temperature of 36.2 C.

28. A female nurse is interviewing a man who has recently immigrated. During the course of the interview, he leans forward and then finally moves his chair close enough that his knees are nearly touching the nurses knees. The nurse begins to feel uncomfortable with his proximity. Which statement most closely reflects what the nurse should do next? a. The nurse should try to relax; these behaviors are culturally appropriate for this person. b. The nurse should discreetly move his or her chair back until the distance is more comfortable, and then continue with the interview. c. These behaviors are indicative of sexual aggression, and the nurse should confront this person about his behaviors. d. The nurse should laugh but tell him that he or she is uncomfortable with his proximity and ask him to move away.

a. The nurse should try to relax; these behaviors are culturally appropriate for this person. Both the patients and the nurses sense of spatial distance are significant throughout the interview and physical examination, with culturally appropriate distance zones varying widely. Some cultural groups value close physical proximity and may perceive a health care provider who is distancing him or herself as being aloof and unfriendly.

34. During a follow-up visit, the nurse discovers that a patient has not been taking his insulin on a regular basis. The nurse asks, Why havent you taken your insulin? Which statement is an appropriate evaluation of this question? a. This question may place the patient on the defensive. b. This question is an innocent search for information. c. Discussing his behavior with his wife would have been better. d. A direct question is the best way to discover the reasons for his behavior

a. This question may place the patient on the defensive. The adults use of why questions usually implies blame and condemnation and places the person on the defensive. The other statements are not correct.

33. A female patient does not speak English well, and the nurse needs to choose an interpreter. Which of the following would be the most appropriate choice? a. Trained interpreter b. Male family member c. Female family member d. Volunteer college student from the foreign language studies department

a. Trained interpreter Whenever possible, the nurse should use a trained interpreter, preferably one who knows medical terminology. In general, an older, more mature interpreter is preferred to a younger, less experienced one, and the same gender is preferred when possible.

18. To assess a rectal temperature accurately in an adult, the nurse would: a. Use a lubricated blunt tip thermometer. b. Insert the thermometer 2 to 3 inches into the rectum. c. Leave the thermometer in place up to 8 minutes if the patient is febrile. d. Wait 2 to 3 minutes if the patient has recently smoked a cigarette.

a. Use a lubricated blunt tip thermometer. A lubricated rectal thermometer (with a short, blunt tip) is inserted only 2 to 3 cm (1 inch) into the adult rectum and left in place for 2 minutes. Cigarette smoking does not alter rectal temperatures.

1. The nurse is preparing to palpate the thorax and abdomen of a patient. Which of these statements describes the correct technique for this procedure? Select all that apply. a. Warm the hands first before touching the patient. b. For deep palpation, use one long continuous palpation when assessing the liver. c. Start with light palpation to detect surface characteristics. d. Use the fingertips to examine skin texture, swelling, pulsation, and presence of lumps. e. Identify any tender areas, and palpate them last. f. Use the palms of the hands to assess temperature of the skin.

a. Warm the hands first before touching the patient. c. Start with light palpation to detect surface characteristics. d. Use the fingertips to examine skin texture, swelling, pulsation, and presence of lumps. e. Identify any tender areas, and palpate them last. The hands should always be warmed before beginning palpation. Intermittent pressure rather than one long continuous palpation is used; any tender areas are identified and palpated last. Fingertips are used to examine skin texture, swelling, pulsation, and the presence of lumps. The dorsa (backs) of the hands are used to assess skin temperature because the skin on the dorsa is thinner than on the palms.

18. As the nurse enters a patients room, the nurse finds her crying. The patient states that she has just found out that the lump in her breast is cancer and says, Im so afraid of, um, you know. The nurses most therapeutic response would be to say in a gentle manner: a. Youre afraid you might lose your breast? b. No, Im not sure what you are talking about. c. Ill wait here until you get yourself under control, and then we can talk. d. I can see that you are very upset. Perhaps we should discuss this later.

a. Youre afraid you might lose your breast? Reflection echoes the patients words, repeating part of what the person has just said. Reflection can also help express the feelings behind a persons words.

10. When assessing an older adult, which vital sign changes occur with aging? a. Increase in pulse rate b. Widened pulse pressure c. Increase in body temperature d. Decrease in diastolic blood pressure

b. Widened pulse pressure With aging, the nurse keeps in mind that the systolic blood pressure increases, leading to widened pulse pressure. With many older people, both the systolic and diastolic pressures increase. The pulse rate and temperature do not increase.

3. The nurse is conducting a class on alcohol and the effects of alcohol on the body. How many standard drinks (each containing 14 grams of alcohol) per day in men are associated with increased deaths from cirrhosis, cancers of the mouth, esophagus, and injuries? a. 2 b. 4 c. 6 d. 8

b. 4 In men, alcohol consumption of at least four standard drinks per day is associated with increased deaths from liver cirrhosis, cancers of the mouth, esophagus and other areas, and deaths from injuries and other external causes.

32. The nurse has collected the following information on a patient: palpated blood pressure180 mm Hg; auscultated blood pressure170/100 mm Hg; apical pulse60 beats per minute; radial pulse70 beats per minute. What is the patients pulse pressure? a. 10 b. 70 c. 80 d. 100

b. 70 Pulse pressure is the difference between systolic and diastolic blood pressure (170 100 = 70) and reflects the stroke volume.

40. During an examination of a patients abdomen, the nurse notes that the abdomen is rounded and firm to the touch. During percussion, the nurse notes a drumlike quality of the sounds across the quadrants. This type of sound indicates: a. Constipation. b. Air-filled areas. c. Presence of a tumor. d. Presence of dense organs.

b. Air-filled areas. A musical or drumlike sound (tympany) is heard when percussion occurs over an air-filled viscus, such as the stomach or intestines.

15. In response to a question regarding the use of alcohol, a patient asks the nurse why the nurse needs to know. What is the reason for needing this information? a. This information is necessary to determine the patients reliability. b. Alcohol can interact with all medications and can make some diseases worse. c. The nurse needs to be able to teach the patient about the dangers of alcohol use. d. This information is not necessary unless a drinking problem is obvious.

b. Alcohol can interact with all medications and can make some diseases worse. Alcohol adversely interacts with all medications and is a factor in many social problems such as child or sexual abuse, automobile accidents, and assaults; alcohol also contributes to many illnesses and disease processes. Therefore, assessing for signs of hazardous alcohol use is important. The other options are not correct.

14. The nurse has used interpretation regarding a patients statement or actions. After using this technique, it would be best for the nurse to: a. Apologize, because using interpretation can be demeaning for the patient. b. Allow time for the patient to confirm or correct the inference. c. Continue with the interview as though nothing has happened. d. Immediately restate the nurses conclusion on the basis of the patients nonverbal response.

b. Allow time for the patient to confirm or correct the inference. Interpretation is not based on direct observation as is confrontation, but it is based on ones inference or conclusion. The nurse risks making the wrong inference. If this is the case, then the patient will correct it. However, even if the inference is correct, interpretation helps prompt further discussion of the topic.

13. The nurse is teaching a class on basic assessment skills. Which of these statements is true regarding the stethoscope and its use? a. Slope of the earpieces should point posteriorly (toward the occiput). b. Although the stethoscope does not magnify sound, it does block out extraneous room noise. c. Fit and quality of the stethoscope are not as important as its ability to magnify sound. d. Ideal tubing length should be 22 inches to dampen the distortion of sound.

b. Although the stethoscope does not magnify sound, it does block out extraneous room noise. The stethoscope does not magnify sound, but it does block out extraneous room sounds. The slope of the earpieces should point forward toward the examiners nose. Long tubing will distort sound. The fit and quality of the stethoscope are both important.

39. The nurse is conducting a health fair for older adults. Which statement is true regarding vital sign measurements in aging adults? a. The pulse is more difficult to palpate because of the stiffness of the blood vessels. b. An increased respiratory rate and a shallower inspiratory phase are expected findings. c. A decreased pulse pressure occurs from changes in the systolic and diastolic blood pressures. d. Changes in the bodys temperature regulatory mechanism leave the older person more likely to develop a fever.

b. An increased respiratory rate and a shallower inspiratory phase are expected findings. Aging causes a decrease in vital capacity and decreased inspiratory reserve volume. The examiner may notice a shallower inspiratory phase and an increased respiratory rate. An increase in the rigidity of the arterial walls makes the pulse actually easier to palpate. Pulse pressure is widened in older adults, and changes in the body temperature regulatory mechanism leave the older person less likely to have fever but at a greater risk for hypothermia.

30. The nurse is incorporating a persons spiritual values into the health history. Which of these questions illustrates the community portion of the FICA (faith and belief, importance and influence, community, and addressing or applying in care) questions? a. Do you believe in God? b. Are you a part of any religious or spiritual congregation? c. Do you consider yourself to be a religious or spiritual person? d. How does your religious faith influence the way you think about your health?

b. Are you a part of any religious or spiritual congregation? The community is assessed when the nurse asks whether a person is part of a religious or spiritual community or congregation. The other areas assessed are faith, influence, and addressing any religious or spiritual issues or concerns.

25. The nurse is performing a functional assessment on an 82-year-old patient who recently had a stroke. Which of these questions would be most important to ask? a. Do you wear glasses? b. Are you able to dress yourself? c. Do you have any thyroid problems? d. How many times a day do you have a bowel movement?

b. Are you able to dress yourself? Functional assessment measures how a person manages day-to-day activities. For the older person, the meaning of health becomes those activities that they can or cannot do. The other responses do not relate to functional assessment.

26. The nurse is examining an infant and prepares to elicit the Moro reflex at which time during the examination? a. When the infant is sleeping b. At the end of the examination c. Before auscultation of the thorax d. Halfway through the examination

b. At the end of the examination The Moro or startle reflex is elicited at the end of the examination because it may cause the infant to cry.

24. A 16-year-old boy has just been admitted to the unit for overnight observation after being in an automobile accident. What is the nurses best approach to communicating with him? a. Use periods of silence to communicate respect for him. b. Be totally honest with him, even if the information is unpleasant. c. Tell him that everything that is discussed will be kept totally confidential. d. Use slang language when possible to help him open up.

b. Be totally honest with him, even if the information is unpleasant. Successful communication with an adolescent is possible and can be rewarding. The guidelines are simple. The first consideration is ones attitude, which must be one of respect. Second, communication must be totally honest. An adolescents intuition is highly tuned and can detect phoniness or the withholding of information. Always tell him or her the truth.

39. While auscultating heart sounds, the nurse hears a murmur. Which of these instruments should be used to assess this murmur? a. Electrocardiogram b. Bell of the stethoscope c. Diaphragm of the stethoscope d. Palpation with the nurses palm of the hand

b. Bell of the stethoscope The bell of the stethoscope is best for soft, low-pitched sounds such as extra heart sounds or murmurs. The diaphragm of the stethoscope is best used for high-pitched sounds such as breath, bowel, and normal heart sounds.

12. A patient has suddenly developed shortness of breath and appears to be in significant respiratory distress. After calling the physician and placing the patient on oxygen, which of these actions is the best for the nurse to take when further assessing the patient? a. Count the patients respirations. b. Bilaterally percuss the thorax, noting any differences in percussion tones. c. Call for a chest x-ray study, and wait for the results before beginning an assessment. d. Inspect the thorax for any new masses and bleeding associated with respirations.

b. Bilaterally percuss the thorax, noting any differences in percussion tones. Percussion is always available, portable, and offers instant feedback regarding changes in underlying tissue density, which may yield clues of the patients physical status.

14. The nurse is reviewing theories of illness. The germ theory, which states that microscopic organisms such as bacteria and viruses are responsible for specific disease conditions, is a basic belief of which theory of illness? a. Holistic b. Biomedical c. Naturalistic d. Magicoreligious

b. Biomedical Among the biomedical explanations for disease is the germ theory, which states that microscopic organisms such as bacteria and viruses are responsible for specific disease conditions. The naturalistic, or holistic, perspective holds that the forces of nature must be kept in natural balance. The magicoreligious perspective holds that supernatural forces dominate and cause illness or health.

10. When assessing a patients pain, the nurse knows that an example of visceral pain would be: a. Hip fracture. b. Cholecystitis. c. Second-degree burns. d. Pain after a leg amputation.

b. Cholecystitis. Visceral pain originates from the larger interior organs, such as the gallbladder, liver, or kidneys.

35. A patient repeats, I feel hot. Hot, cot, rot, tot, got. Im a spot. The nurse documents this as an illustration of: a. Blocking b. Clanging c. Echolalia d. Neologism

b. Clanging Clanging is word choice based on sound, not meaning, and includes nonsense rhymes and puns. (See Table 5-6 for the definitions of the other terms.)

36. During an interview, the nurse notes that the patient gets up several times to wash her hands even though they are not dirty. This behavior is an example of: a. Social phobia b. Compulsive disorder c. Generalized anxiety disorder d. Posttraumatic stress disorder

b. Compulsive disorder Repetitive behaviors, such as handwashing, are behaviors that the person feels driven to perform in response to an obsession. The behaviors are aimed at preventing or reducing distress or preventing some dreaded event or situation.

5. A 1-month-old infant has a head measurement of 34 cm and has a chest circumference of 32 cm. Based on the interpretation of these findings, the nurse would: a. Refer the infant to a physician for further evaluation. b. Consider these findings normal for a 1-month-old infant. c. Expect the chest circumference to be greater than the head circumference. d. Ask the parent to return in 2 weeks to re-evaluate the head and chest circumferences.

b. Consider these findings normal for a 1-month-old infant. The newborns head measures approximately 32 to 38 cm and is approximately 2 cm larger than the chest circumference. Between 6 months and 2 years, both measurements are approximately the same, and after age 2 years, the chest circumference is greater than the head circumference.

13. In the majority culture of America, coughing, sweating, and diarrhea are symptoms of an illness. For some individuals of Mexican-American origin, however, these symptoms are a normal part of living. The nurse recognizes that this difference is true, probably because Mexican-Americans: a. Have less efficient immune systems and are often ill. b. Consider these symptoms part of normal living, not symptoms of ill health. c. Come from Mexico, and coughing is normal and healthy there. d. Are usually in a lower socioeconomic group and are more likely to be sick.

b. Consider these symptoms part of normal living, not symptoms of ill health. The nurse needs to identify the meaning of health to the patient, remembering that concepts are derived, in part, from the way in which members of the cultural group define health.

21. When assessing the pulse of a 6-year-old boy, the nurse notices that his heart rate varies with his respiratory cycle, speeding up at the peak of inspiration and slowing to normal with expiration. The nurses next action would be to: a. Immediately notify the physician. b. Consider this finding normal in children and young adults. c. Check the childs blood pressure, and note any variation with respiration. d. Document that this child has bradycardia, and continue with the assessment.

b. Consider this finding normal in children and young adults. Sinus arrhythmia is commonly found in children and young adults. During the respiratory cycle, the heart rate varies, speeding up at the peak of inspiration and slowing to normal with expiration.

6. When reviewing the use of alcohol by older adults, the nurse notes that older adults have several characteristics that can increase the risk of alcohol use. Which would increase the bioavailability of alcohol in the blood for longer periods in the older adult? a. Increased muscle mass b. Decreased liver and kidney functioning c. Decreased blood pressure d. Increased cardiac output

b. Decreased liver and kidney functioning Decreased liver and kidney functioning increases the bioavailability of alcohol in the blood for longer periods. Aging people experience decreased muscle mass (not increased), which also increases the alcohol concentration in the blood because the alcohol is distributed to less tissue over time. Blood pressure and cardiac output are not factors regarding bioavailability.

22. A 21-year-old woman has been on a low-protein liquid diet for the past 2 months. She has had adequate intake of calories and appears well nourished. After further assessment, what would the nurse expect to find? a. Poor skin turgor b. Decreased serum albumin c. Increased lymphocyte count d. Triceps skinfold less than standard

b. Decreased serum albumin Kwashiorkor (protein malnutrition) is due to diets that may be high in calories but contain little or no protein (e.g., low-protein liquid diets, fad diets, and long-term use of dextrose-containing intravenous fluids). The serum albumin would be less than 3.5 g/dL.

30. The nurse will perform a palpated pressure before auscultating blood pressure. The reason for this is to: a. More clearly hear the Korotkoff sounds. b. Detect the presence of an auscultatory gap. c. Avoid missing a falsely elevated blood pressure. d. More readily identify phase IV of the Korotkoff sounds.

b. Detect the presence of an auscultatory gap. Inflation of the cuff 20 to 30 mm Hg beyond the point at which a palpated pulse disappears will avoid missing an auscultatory gap, which is a period when the Korotkoff sounds disappear during auscultation.

12. The nurse is conducting a heritage assessment. Which question is most appropriate for this assessment? a. What is your religion? b. Do you mostly participate in the religious traditions of your family? c. Do you smoke? d. Do you have a history of heart disease?

b. Do you mostly participate in the religious traditions of your family? Asking questions about participation in the religious traditions of family enables the nurse to assess a persons heritage. Simply asking about ones religion, smoking history, or health history does not reflect heritage.

3. The nurse is assessing a patients skin during an office visit. What part of the hand and technique should be used to best assess the patients skin temperature? a. Fingertips; they are more sensitive to small changes in temperature. b. Dorsal surface of the hand; the skin is thinner on this surface than on the palms. c. Ulnar portion of the hand; increased blood supply in this area enhances temperature sensitivity. d. Palmar surface of the hand; this surface is the most sensitive to temperature variations because of its increased nerve supply in this area.

b. Dorsal surface of the hand; the skin is thinner on this surface than on the palms. The dorsa (backs) of the hands and fingers are best for determining temperature because the skin is thinner on the dorsal surfaces than on the palms. Fingertips are best for fine, tactile discrimination. The other responses are not useful for palpation.

25. The nurse keeps in mind that the most important reason to share information and to offer brief teaching while performing the physical examination is to help the: a. Examiner feel more comfortable and to gain control of the situation. b. Examiner to build rapport and to increase the patients confidence in him or her. c. Patient understand his or her disease process and treatment modalities. d. Patient identify questions about his or her disease and the potential areas of patient education.

b. Examiner to build rapport and to increase the patients confidence in him or her. Sharing information builds rapport and increases the patients confidence in the examiner. It also gives the patient a little more control in a situation during which feeling completely helpless is often present.

1. The nurse is conducting an interview with a woman who has recently learned that she is pregnant and who has come to the clinic today to begin prenatal care. The woman states that she and her husband are excited about the pregnancy but have a few questions. She looks nervously at her hands during the interview and sighs loudly. Considering the concept of communication, which statement does the nurse know to be most accurate? The woman is: a. Excited about her pregnancy but nervous about the labor. b. Exhibiting verbal and nonverbal behaviors that do not match. c. Excited about her pregnancy, but her husband is not and this is upsetting to her. d. Not excited about her pregnancy but believes the nurse will negatively respond to her if she states this.

b. Exhibiting verbal and nonverbal behaviors that do not match. Communication is all behaviors, conscious and unconscious, verbal and nonverbal. All behaviors have meaning. Her behavior does not imply that she is nervous about labor, upset by her husband, or worried about the nurses response.

31. The nurse is assessing the body weight as a percentage of ideal body weight on an adolescent patient who was admitted for suspected anorexia nervosa. The patients usual weight was 125 pounds, but today she weighs 98 pounds. The nurse calculates the patients ideal body weight and concludes that the patient is: a. Experiencing mild malnutrition. b. Experiencing moderate malnutrition. c. Experiencing severe malnutrition. d. Still within expected parameters with her current weight.

b. Experiencing moderate malnutrition. By dividing her current weight by her usual weight and then multiplying by 100, a percentage of 78.4% is obtained, which means that her current weight is 78.4% of her ideal body weight. A current weight of 80% to 90% of ideal weight suggests mild malnutrition; a current weight of 70% to 80% of ideal weight suggests moderate malnutrition; a current weight of less than 70% of ideal weight suggests severe malnutrition.

11. A woman is discussing the problems she is having with her 2-year-old son. She says, He wont go to sleep at night, and during the day he has several fits. I get so upset when that happens. The nurses best verbal response would be: a. Go on, Im listening. b. Fits? Tell me what you mean by this. c. Yes, it can be upsetting when a child has a fit. d. Dont be upset when he has a fit; every 2 year old has fits.

b. Fits? Tell me what you mean by this. The nurse should use clarification when the persons word choice is ambiguous or confusing (e.g., Tell me what you mean by fits.). Clarification is also used to summarize the persons words or to simplify the words to make them clearer; the nurse should then ask if he or she is on the right track.

5. The nurse is preparing to conduct a mental status examination. Which statement is true regarding the mental status examination? a. A patients family is the best resource for information about the patients coping skills. b. Gathering mental status information during the health history interview is usually sufficient. c. Integrating the mental status examination into the health history interview takes an enormous amount of extra time. d. To get a good idea of the patients level of functioning, performing a complete mental status examination is usually necessary.

b. Gathering mental status information during the health history interview is usually sufficient. The full mental status examination is a systematic check of emotional and cognitive functioning. The steps described, however, rarely need to be taken in their entirety. Usually, one can assess mental status through the context of the health history interview.

32. The nurse is preparing to examine a 4-year-old child. Which action is appropriate for this age group? a. Explain the procedures in detail to alleviate the childs anxiety. b. Give the child feedback and reassurance during the examination. c. Do not ask the child to remove his or her clothes because children at this age are usually very private. d. Perform an examination of the ear, nose, and throat first, and then examine the thorax and abdomen.

b. Give the child feedback and reassurance during the examination. With preschool children, short, simple explanations should be used. Children at this age are usually willing to undress. An examination of the head should be performed last. During the examination, needed feedback and reassurance should be given to the preschooler.

17. During an assessment of a patients family history, the nurse constructs a genogram. Which statement best describes a genogram? a. List of diseases present in a persons near relatives b. Graphic family tree that uses symbols to depict the gender, relationship, and age of immediate family members c. Drawing that depicts the patients family members up to five generations back d. Description of the health of a persons children and grandchildren

b. Graphic family tree that uses symbols to depict the gender, relationship, and age of immediate family members A genogram (or pedigree) is a graphic family tree that uses symbols to depict the gender, relationship, and age of immediate blood relatives in at least three generations (parents, grandparents, siblings). The other options do not describe a genogram.

7. A female patient tells the nurse that she has had six pregnancies, with four live births at term and two spontaneous abortions. Her four children are still living. How would the nurse record this information? a. P-6, B-4, (S)Ab-2 b. Grav 6, Term 4, (S)Ab-2, Living 4 c. Patient has had four living babies. d. Patient has been pregnant six times.

b. Grav 6, Term 4, (S)Ab-2, Living 4 Obstetric history includes the number of pregnancies (gravidity), number of deliveries in which the fetus reached term (term), number of preterm pregnancies (preterm), number of incomplete pregnancies (abortions), and number of children living (living). This is recorded: Grav _____ Term _____ Preterm _____ Ab _____ Living _____. For any incomplete pregnancies, the duration is recorded and whether the pregnancy resulted in a spontaneous (S) or an induced (I) abortion.

22. When performing a physical examination, safety must be considered to protect the examiner and the patient against the spread of infection. Which of these statements describes the most appropriate action the nurse should take when performing a physical examination? a. Washing ones hands after removing gloves is not necessary, as long as the gloves are still intact. b. Hands are washed before and after every physical patient encounter. c. Hands are washed before the examination of each body system to prevent the spread of bacteria from one part of the body to another. d. Gloves are worn throughout the entire examination to demonstrate to the patient concern regarding the spread of infectious diseases.

b. Hands are washed before and after every physical patient encounter. The nurse should wash his or her hands before and after every physical patient encounter; after contact with blood, body fluids, secretions, and excretions; after contact with any equipment contaminated with body fluids; and after removing gloves. Hands should be washed after gloves have been removed, even if the gloves appear to be intact. Gloves should be worn when potential contact with any body fluids is present.

46. Which of these specific measurements is the best index of a childs general health? a. Vital signs b. Height and weight c. Head circumference d. Chest circumference

b. Height and weight Physical growth, measured by height and weight, is the best index of a childs general health.

8. The nurse recognizes that an example of a person who is heritage consistent would be a: a. Woman who has adapted her clothing to the clothing style of her new country. b. Woman who follows the traditions that her mother followed regarding meals. c. Man who is not sure of his ancestors country of origin. d. Child who is not able to speak his parents native language.

b. Woman who follows the traditions that her mother followed regarding meals. Someone who is heritage consistent lives a lifestyle that reflects his or her traditional heritage, not the norms and customs of the new country.

10. A patient is brought to the emergency department. He is restless, has dilated pupils, is sweating, has a runny nose and tearing eyes, and complains of muscle and joint pains. His girlfriend thinks he has influenza, but she became concerned when his temperature went up to 39.4 C. She admits that he has been a heavy drug user, but he has been trying to stop on his own. The nurse suspects that the patient is experiencing withdrawal symptoms from which substance? a. Alcohol b. Heroin c. Crack cocaine d. Sedatives

b. Heroin Withdrawal symptoms of opiates, such as heroin, are similar to the clinical picture of influenza and include symptoms such as dilated pupils, lacrimation, runny nose, tachycardia, fever, restlessness, muscle and joint pains, and other symptoms. (Withdrawal symptoms from alcohol, cocaine, and sedatives are described in Table 6-7.)

12. When assessing the intensity of a patients pain, which question by the nurse is appropriate? a. What makes your pain better or worse? b. How much pain do you have now? c. How does pain limit your activities? d. What does your pain feel like?

b. How much pain do you have now? Asking the patient how much pain do you have? is an assessment of the intensity of a patients pain; various intensity scales can be used. Asking what makes ones pain better or worse assesses alleviating or aggravating factors. Asking whether pain limits ones activities assesses the degree of impairment and quality of life. Asking what does your pain feel like assesses the quality of pain.

27. During a prenatal check, a patient begins to cry as the nurse asks her about previous pregnancies. She states that she is remembering her last pregnancy, which ended in miscarriage. The nurses best response to her crying would be: a. Im so sorry for making you cry! b. I can see that you are sad remembering this. It is all right to cry. c. Why dont I step out for a few minutes until youre feeling better? d. I can see that you feel sad about this; why dont we talk about something else?

b. I can see that you are sad remembering this. It is all right to cry. A beginning examiner usually feels horrified when the patient starts crying. When the nurse says something that makes the person cry, the nurse should not think he or she has hurt the person. The nurse has simply hit on an important topic; therefore, moving on to a new topic is essential. The nurse should allow the person to cry and to express his or her feelings fully. The nurse can offer a tissue and wait until the crying subsides to talk.

21. In performing an assessment on a 49-year-old woman who has imbalanced nutrition as a result of dysphagia, which data would the nurse expect to find? a. Increase in hair growth b. Inadequate nutrient food intake c. Weight 10% to 20% over ideal d. Sore, inflamed buccal cavity

b. Inadequate nutrient food intake Dysphagia, or impaired swallowing, interferes with adequate nutrient intake.

10. The review of systems provides the nurse with: a. Physical findings related to each system. b. Information regarding health promotion practices. c. An opportunity to teach the patient medical terms. d. Information necessary for the nurse to diagnose the patients medical problem.

b. Information regarding health promotion practices. The purposes of the review of systems are to: (1) evaluate the past and current health state of each body system, (2) double check facts in case any significant data were omitted in the present illness section, and (3) evaluate health promotion practices.

When performing a physical assessment, the first technique the nurse will always use is: a. Palpation. b. Inspection. c. Percussion. d. Auscultation.

b. Inspection. The skills requisite for the physical examination are inspection, palpation, percussion, and auscultation. The skills are performed one at a time and in this order (with the exception of the abdominal assessment, during which auscultation takes place before palpation and percussion). The assessment of each body system begins with inspection. A focused inspection takes time and yields a surprising amount of information.

35. The nurse is nearing the end of an interview. Which statement is appropriate at this time? a. Did we forget something? b. Is there anything else you would like to mention? c. I need to go on to the next patient. Ill be back. d. While Im here, lets talk about your upcoming surgery.

b. Is there anything else you would like to mention? This question offers the person a final opportunity for self-expression. No new topic should be introduced. The other questions are not appropriate.

23. The nurse is reviewing concepts of cultural aspects of pain. Which statement is true regarding pain? a. All patients will behave the same way when in pain. b. Just as patients vary in their perceptions of pain, so will they vary in their expressions of pain. c. Cultural norms have very little to do with pain tolerance, because pain tolerance is always biologically determined. d. A patients expression of pain is largely dependent on the amount of tissue injury associated with the pain.

b. Just as patients vary in their perceptions of pain, so will they vary in their expressions of pain. In addition to expecting variations in pain perception and tolerance, the nurse should expect variations in the expression of pain. It is well known that individuals turn to their social environment for validation and comparison. The other statements are incorrect.

The nurse is reviewing the development of culture. Which statement is correct regarding the development of ones culture? Culture is: a. Genetically determined on the basis of racial background. b. Learned through language acquisition and socialization. c. A nonspecific phenomenon and is adaptive but unnecessary. d. Biologically determined on the basis of physical characteristics.

b. Learned through language acquisition and socialization. Culture is learned from birth through language acquisition and socialization. It is not biologically or genetically determined and is learned by the individual

16. Which of these actions illustrates the correct technique the nurse should use when assessing oral temperature with a mercury thermometer? a. Wait 30 minutes if the patient has ingested hot or iced liquids. b. Leave the thermometer in place 3 to 4 minutes if the patient is afebrile. c. Place the thermometer in front of the tongue, and ask the patient to close his or her lips. d. Shake the mercury-in-glass thermometer down to below 36.6 C before taking the temperature.

b. Leave the thermometer in place 3 to 4 minutes if the patient is afebrile. The thermometer should be left in place 3 to 4 minutes if the person is afebrile and up to 8 minutes if the person is febrile. The nurse should wait 15 minutes if the person has just ingested hot or iced liquids and 2 minutes if he or she has just smoked.

14. The nurse is reviewing the nutritional assessment of an 82-year-old patient. Which of these factors will most likely affect the nutritional status of an older adult? a. Increase in taste and smell b. Living alone on a fixed income c. Change in cardiovascular status d. Increase in gastrointestinal motility and absorption

b. Living alone on a fixed income Socioeconomic conditions frequently affect the nutritional status of the aging adult; these factors should be closely evaluated. Physical limitations, income, and social isolation are frequent problems that interfere with the acquisition of a balanced diet. A decrease in taste and smell and decreased gastrointestinal motility and absorption occur with aging. Cardiovascular status is not a factor that affects an older adults nutritional status.

19. As part of the health history of a 6-year-old boy at a clinic for a sports physical examination, the nurse reviews his immunization record and notes that his last measles-mumps-rubella (MMR) vaccination was at 15 months of age. What recommendation should the nurse make? a. No further MMR immunizations are needed. b. MMR vaccination needs to be repeated at 4 to 6 years of age. c. MMR immunization needs to be repeated every 4 years until age 21 years. d. A recommendation cannot be made until the physician is consulted.

b. MMR vaccination needs to be repeated at 4 to 6 years of age. Because of recent outbreaks of measles across the United States, the American Academy of Pediatrics (2006) recommends two doses of the MMR vaccine, one at 12 to 15 months of age and one at age 4 to 6 years.

16. A pregnant woman states, I just know labor will be so painful that I wont be able to stand it. I know it sounds awful, but I really dread going into labor. The nurse responds by stating, Oh, dont worry about labor so much. I have been through it, and although it is painful, many good medications are available to decrease the pain. Which statement is true regarding this response? The nurses reply was a: a. Therapeutic response. By sharing something personal, the nurse gives hope to this woman. b. Nontherapeutic response. By providing false reassurance, the nurse actually cut off further discussion of the womans fears. c. Therapeutic response. By providing information about the medications available, the nurse is giving information to the woman. d. Nontherapeutic response. The nurse is essentially giving the message to the woman that labor cannot be tolerated without medication.

b. Nontherapeutic response. By providing false reassurance, the nurse actually cut off further discussion of the womans fears. By providing false assurance or reassurance, this courage builder relieves the womans anxiety and gives the nurse the false sense of having provided comfort. However, for the woman, providing false assurance or reassurance actually closes off communication, trivializes her anxiety, and effectively denies any further talk of it.

22. Symptoms, such as pain, are often influenced by a persons cultural heritage. Which of the following is a true statement regarding pain? a. Nurses attitudes toward their patients pain are unrelated to their own experiences with pain. b. Nurses need to recognize that many cultures practice silent suffering as a response to pain. c. A nurses area of clinical practice will most likely determine his or her assessment of a patients pain. d. A nurses years of clinical experience and current position are strong indicators of his or her

b. Nurses need to recognize that many cultures practice silent suffering as a response to pain. Silent suffering is a potential response to pain in many cultures. The nurses assessment of pain needs to be embedded in a cultural context. The other responses are not correct.

6. The nurse would use bimanual palpation technique in which situation? a. Palpating the thorax of an infant b. Palpating the kidneys and uterus c. Assessing pulsations and vibrations d. Assessing the presence of tenderness and pain

b. Palpating the kidneys and uterus Bimanual palpation requires the use of both hands to envelop or capture certain body parts or organs such as the kidneys, uterus, or adnexa. The other situations are not appropriate for bimanual palpation.

3. A patients weekly blood pressure readings for 2 months have ranged between 124/84 mm Hg and 136/88 mm Hg, with an average reading of 126/86 mm Hg. The nurse knows that this blood pressure falls within which blood pressure category? a. Normal blood pressure b. Prehypertension c. Stage 1 hypertension d. Stage 2 hypertension

b. Prehypertension According to the Seventh Report of the Joint National Committee (JNC 7) guidelines, prehypertension blood pressure readings are systolic readings of 120 to 139 mm Hg or diastolic readings of 50 to 89 mm Hg.

31. With which of these patients would it be most appropriate for the nurse to use games during the assessment, such as having the patient blow out the light on the penlight? a. Infant b. Preschool child c. School-age child d. Adolescent

b. Preschool child When assessing preschool children, using games or allowing them to play with the equipment to reduce their fears can be helpful. Such games are not appropriate for the other age groups.

2. When the nurse is evaluating the reliability of a patients responses, which of these statements would be correct? The patient: a. Has a history of drug abuse and therefore is not reliable. b. Provided consistent information and therefore is reliable. c. Smiled throughout interview and therefore is assumed reliable. d. Would not answer questions concerning stress and therefore is not reliable.

b. Provided consistent information and therefore is reliable. A reliable person always gives the same answers, even when questions are rephrased or are repeated later in the interview. The other statements are not correct.

11. The nurse is examining a patient who is complaining of feeling cold. Which is a mechanism of heat loss in the body? a. Exercise b. Radiation c. Metabolism d. Food digestion

b. Radiation The body maintains a steady temperature through a thermostat or feedback mechanism, which is regulated in the hypothalamus of the brain. The hypothalamus regulates heat production from metabolism, exercise, food digestion, and external factors with heat loss through radiation, evaporation of sweat, convection, and conduction.

33. The nurse is reviewing concepts related to ones heritage and beliefs. The belief in divine or superhuman power(s) to be obeyed and worshipped as the creator(s) and ruler(s) of the universe is known as: a. Culture. b. Religion. c. Ethnicity. d. Spirituality.

b. Religion. Religion is defined as an organized system of beliefs concerning the cause, nature, and purpose of the universe, especially belief in or the worship of God or gods. Spirituality is born out of each persons unique life experiences and his or her personal efforts to find purpose and meaning in life. Ethnicity pertains to a social group within the social system that claims to possess variable traits, such as a common geographic origin, religion, race, and others.

11. The nurse is reviewing aspects of substance abuse in preparation for a seminar. Which of these statements illustrates the concept of tolerance to an illicit substance? The person: a. Has a physiologic dependence on a substance. b. Requires an increased amount of the substance to produce the same effect. c. Requires daily use of the substance to function and is unable to stop using it. d. Experiences a syndrome of physiologic symptoms if the substance is not used.

b. Requires an increased amount of the substance to produce the same effect. The concept of tolerance to a substance indicates that the person requires an increased amount of the substance to produce the same effect. Abuse indicates that the person needs to use the substance daily to function, and the person is unable to stop using it. Dependence is an actual physiologic dependence on the substance. Withdrawal occurs when cessation of the substance produces a syndrome of physiologic symptoms.

28. During an assessment of a patient who has been homeless for several years, the nurse notices that his tongue is magenta in color, which is an indication of a deficiency in what mineral and/or vitamin? a. Iron b. Riboflavin c. Vitamin D and calcium d. Vitamin C

b. Riboflavin Magenta tongue is a sign of riboflavin deficiency. In contrast, a pale tongue is probably attributable to iron deficiency. Vitamin D and calcium deficiencies cause osteomalacia in adults, and a vitamin C deficiency causes scorbutic gums.

5. A mother and her 13-year-old daughter express their concern related to the daughters recent weight gain and her increase in appetite. Which of these statements represents information the nurse should discuss with them? a. Dieting and exercising are necessary at this age. b. Snacks should be high in protein, iron, and calcium. c. Teenagers who have a weight problem should not be allowed to snack. d. A low-calorie diet is important to prevent the accumulation of fat.

b. Snacks should be high in protein, iron, and calcium. After a period of slow growth in late childhood, adolescence is characterized by rapid physical growth and endocrine and hormonal changes. Caloric and protein requirements increase to meet this demand. Because of bone growth and increasing muscle mass (and, in girls, the onset of menarche), calcium and iron requirements also increase.

9. The nurse has completed an assessment on a patient who came to the clinic for a leg injury. As a result of the assessment, the nurse has determined that the patient has at-risk alcohol use. Which action by the nurse is most appropriate at this time? a. Record the results of the assessment, and notify the physician on call. b. State, You are drinking more than is medically safe. I strongly recommend that you quit drinking, and Im willing to help you. c. State, It appears that you may have a drinking problem. Here is the telephone number of our local Alcoholics Anonymous chapter. d. Give the patient information about a local rehabilitation clinic.

b. State, You are drinking more than is medically safe. I strongly recommend that you quit drinking, and Im willing to help you. If an assessment has determined that the patient has at-risk drinking behavior, then the nurse should give a short but clear statement of assistance and concern. Simply giving out a telephone number or referral to agencies may not be enough.

22. A mother brings her 28-month-old daughter into the clinic for a well-child visit. At the beginning of the visit, the nurse focuses attention away from the toddler, but as the interview progresses, the toddler begins to warm up and is smiling shyly at the nurse. The nurse will be most successful in interacting with the toddler if which is done next? a. Tickle the toddler, and get her to laugh. b. Stoop down to her level, and ask her about the toy she is holding. c. Continue to ignore her until it is time for the physical examination. d. Ask the mother to leave during the examination of the toddler, because toddlers often fuss less if their parent is not in view.

b. Stoop down to her level, and ask her about the toy she is holding. Although most of the communication is with the parent, the nurse should not completely ignore the child. Making contact will help ease the toddler later during the physical examination. The nurse should begin by asking about the toys the child is playing with or about a special doll or teddy bear brought from home. Does your doll have a name? or What can your truck do? Stoop down to meet the child at his or her eye level.

18. An individual who takes the magicoreligious perspective of illness and disease is likely to believe that his or her illness was caused by: a. Germs and viruses. b. Supernatural forces. c. Eating imbalanced foods. d. An imbalance within his or her spiritual nature.

b. Supernatural forces. The basic premise of the magicoreligious perspective is that the world is seen as an arena in which supernatural forces dominate. The fate of the world and those in it depends on the actions of supernatural forces for good or evil. The other answers do not reflect the magicoreligious perspective.

2. The nurse is preparing to perform a physical assessment. Which statement is true about the physical assessment? The inspection phase: a. Usually yields little information. b. Takes time and reveals a surprising amount of information. c. May be somewhat uncomfortable for the expert practitioner. d. Requires a quick glance at the patients body systems before proceeding with palpation.

b. Takes time and reveals a surprising amount of information. A focused inspection takes time and yields a surprising amount of information. Initially, the examiner may feel uncomfortable, staring at the person without also doing something. A focused assessment is significantly more than a quick glance.

12. During an interview, a woman has answered yes to two of the Abuse Assessment Screen questions. What should the nurse say next? a. I need to report this abuse to the authorities. b. Tell me about this abuse in your relationship. c. So you were abused? d. Do you know what caused this abuse?

b. Tell me about this abuse in your relationship. If a woman answers yes to any of the Abuse Assessment Screen questions, then the nurse should ask questions designed to assess how recent and how serious the abuse was. Asking the woman an open-ended question, such as tell me about this abuse in your relationship is a good way to start.

14. During a mental status assessment, which question by the nurse would best assess a persons judgment? a. Do you feel that you are being watched, followed, or controlled? b. Tell me what you plan to do once you are discharged from the hospital. c. What does the statement, People in glass houses shouldnt throw stones, mean to you? d. What would you do if you found a stamped, addressed envelope lying on the sidewalk?

b. Tell me what you plan to do once you are discharged from the hospital. A person exercises judgment when he or she can compare and evaluate the alternatives in a situation and reach an appropriate course of action. Rather than testing the persons response to a hypothetical situation (as illustrated in the option with the envelope), the nurse should be more interested in the persons judgment about daily or long-term goals, the likelihood of acting in response to delusions or hallucinations, and the capacity for violent or suicidal behavior.

27. A nurse is helping at a health fair at a local mall. When taking blood pressures on a variety of people, the nurse keeps in mind that: a. After menopause, blood pressure readings in women are usually lower than those taken in men. b. The blood pressure of a Black adult is usually higher than that of a White adult of the same age. c. Blood pressure measurements in people who are overweight should be the same as those of people who are at a normal weight. d. A teenagers blood pressure reading will be lower than that of an adult.

b. The blood pressure of a Black adult is usually higher than that of a White adult of the same age. In the United States, a Black adults blood pressure is usually higher than that of a White adult of the same age. The incidence of hypertension is twice as high in Blacks as it is in Whites. After menopause, blood pressure in women is higher than in men; blood pressure measurements in people who are obese are usually higher than in those who are not overweight. Normally, a gradual rise occurs through childhood and into the adult years.

35. A 70-year-old man has a blood pressure of 150/90 mm Hg in a lying position, 130/80 mm Hg in a sitting position, and 100/60 mm Hg in a standing position. How should the nurse evaluate these findings? a. These readings are a normal response and attributable to changes in the patients position. b. The change in blood pressure readings is called orthostatic hypotension. c. The blood pressure reading in the lying position is within normal limits. d. The change in blood pressure readings is considered within normal limits for the patients age.

b. The change in blood pressure readings is called orthostatic hypotension. Orthostatic hypotension is a drop in systolic pressure of more than 20 mm Hg, which occurs with a quick change to a standing position. Aging people have the greatest risk of this problem.

38. A 4-month-old child is at the clinic for a well-baby check-up and immunizations. Which of these actions is most appropriate when the nurse is assessing an infants vital signs? a. The infants radial pulse should be palpated, and the nurse should notice any fluctuations resulting from activity or exercise. b. The nurse should auscultate an apical rate for 1 minute and then assess for any normal irregularities, such as sinus arrhythmia. c. The infants blood pressure should be assessed by using a stethoscope with a large diaphragm piece to hear the soft muffled Korotkoff sounds. d. The infants chest should be observed and the respiratory rate counted for 1 minute; the respiratory pattern may vary significantly.

b. The nurse should auscultate an apical rate for 1 minute and then assess for any normal irregularities, such as sinus arrhythmia. The nurse palpates or auscultates an apical rate with infants and toddlers. The pulse should be counted for 1 full minute to account for normal irregularities, such as sinus arrhythmia. Children younger than 3 years of age have such small arm vessels; consequently, hearing Korotkoff sounds with a stethoscope is difficult. The nurse should use either an electronic blood pressure device that uses oscillometry or a Doppler ultrasound device to amplify the sounds.

23. The nurse is assessing the vital signs of a 20-year-old male marathon runner and documents the following vital signs: temperature36 C; pulse48 beats per minute; respirations14 breaths per minute; blood pressure104/68 mm Hg. Which statement is true concerning these results? a. The patient is experiencing tachycardia. b. These are normal vital signs for a healthy, athletic adult. c. The patients pulse rate is not normalhis physician should be notified. d. On the basis of these readings, the patient should return to the clinic in 1 week.

b. These are normal vital signs for a healthy, athletic adult. In the adult, a heart rate less than 50 beats per minute is called bradycardia, which normally occurs in the welltrained athlete whose heart muscle develops along with the skeletal muscles

11. The nurse suspects abuse when a 10-year-old child is taken to the urgent care center for a leg injury. The best way to document the history and physical findings is to: a. Document what the childs caregiver tells the nurse. b. Use the words the child has said to describe how the injury occurred. c. Record what the nurse observes during the conversation. d. Rely on photographs of the injuries.

b. Use the words the child has said to describe how the injury occurred. When documenting the history and physical findings of suspected child abuse and neglect, use the words the child has said to describe how his or her injury occurred. Remember, the abuser may be accompanying the child.

48. The nurse is counting an infants respirations. Which technique is correct? a. Watching the chest rise and fall b. Watching the abdomen for movement c. Placing a hand across the infants chest d. Using a stethoscope to listen to the breath sounds

b. Watching the abdomen for movement Watching the abdomen for movement is the correct technique because the infants respirations are normally more diaphragmatic than thoracic. The other responses do not reflect correct techniques.

32. The nurse is assessing a new patient who has recently immigrated to the United States. Which question is appropriate to add to the health history? a. Why did you come to the United States? b. When did you come to the United States and from what country? c. What made you leave your native country? d. Are you planning to return to your home?

b. When did you come to the United States and from what country? Biographic data, such as when the person entered the United States and from what country, are appropriate additions to the health history. The other answers do not reflect appropriate questions.

11. A patient tells the nurse that his food simply does not have any taste anymore. The nurses best response would be: a. That must be really frustrating. b. When did you first notice this change? c. My food doesnt always have a lot of taste either. d. Sometimes that happens, but your taste will come back.

b. When did you first notice this change? With changes in appetite, taste, smell, or chewing or swallowing, the examiner should ask about the type of change and when the change occurred. These problems interfere with adequate nutrient intake. The other responses are not correct.

17. During a visit to the clinic, a patient states, The doctor just told me he thought I ought to stop smoking. He doesnt understand how hard Ive tried. I just dont know the best way to do it. What should I do? The nurses most appropriate response in this case would be: a. Id quit if I were you. The doctor really knows what he is talking about. b. Would you like some information about the different ways a person can quit smoking? c. Stopping your dependence on cigarettes can be very difficult. I understand how you feel. d. Why are you confused? Didnt the doctor give you the information about the smoking cessation program we offer?

b. Would you like some information about the different ways a person can quit smoking? Clarification should be used when the persons word choice is ambiguous or confusing. Clarification is also used to summarize the persons words or to simplify the words to make them clearer; the nurse should then ask if he or she is on the right track. The other responses give unwanted advice or do not offer a helpful response.

28. The nurse notices a colleague is preparing to check the blood pressure of a patient who is obese by using a standard-sized blood pressure cuff. The nurse should expect the reading to: a. Yield a falsely low blood pressure. b. Yield a falsely high blood pressure. c. Be the same, regardless of cuff size. d. Vary as a result of the technique of the person performing the assessment.

b. Yield a falsely high blood pressure. Using a cuff that is too narrow yields a falsely high blood pressure because it takes extra pressure to compress the artery.

14. The nurse knows that which statement is true regarding the pain experienced by infants? a. Pain in infants can only be assessed by physiologic changes, such as an increased heart rate. b. The FPS-R can be used to assess pain in infants. c. A procedure that induces pain in adults will also induce pain in the infant. d. Infants feel pain less than do adults.

c. A procedure that induces pain in adults will also induce pain in the infant. If a procedure or disease process causes pain in an adult, then it will also cause pain in an infant. Physiologic changes cannot be exclusively used to confirm or deny pain because other factors, such as medications, fluid status, or stress may cause physiologic changes. The FPS-R can be used starting at age 4 years.

17. How should the nurse perform a triceps skinfold assessment? a. After pinching the skin and fat, the calipers are vertically applied to the fat fold. b. The skin and fat on the front of the patients arm are gently pinched, and then the calipers are applied. c. After applying the calipers, the nurse waits 3 seconds before taking a reading. After repeating the procedure three times, an average is recorded. d. The patient is instructed to stand with his or her back to the examiner and arms folded across the chest. The skin on the forearm is pinched.

c. After applying the calipers, the nurse waits 3 seconds before taking a reading. After repeating the procedure three times, an average is recorded. While holding the skinfold, the lever of the calipers is released. The nurse waits 3 seconds and then takes a reading. This procedure should be repeated three times, and an average of the three skinfold measurements is then recorded.

16. Many Asians believe in the yin/yang theory, which is rooted in the ancient Chinese philosophy of Tao. Which statement most accurately reflects health in an Asian with this belief? a. A person is able to work and produce. b. A person is happy, stable, and feels good. c. All aspects of the person are in perfect balance. d. A person is able to care for others and function socially.

c. All aspects of the person are in perfect balance. Many Asians believe in the yin/yang theory, in which health is believed to exist when all aspects of the person are in perfect balance. The other statements do not describe this theory.

8. A female patient has denied any abuse when answering the Abuse Assessment Screen, but the nurse has noticed some other conditions that are associated with IPV. Examples of such conditions include: a. Asthma b. Confusion. c. Depression. d. Frequent colds.

c. Depression. Depression is one of the conditions that is particularly associated with IPV. Abused women also have been found to have more chronic health problems, such as neurologic, gastrointestinal, and gynecologic symptoms; chronic pain; and symptoms of suicidality and posttraumatic stress disorder.

29. A 2-year-old child has been brought to the clinic for a well-child checkup. The best way for the nurse to begin the assessment is to: a. Ask the parent to place the child on the examining table. b. Have the parent remove all of the childs clothing before the examination. c. Allow the child to keep a security object such as a toy or blanket during the examination. d. Initially focus the interactions on the child, essentially ignoring the parent until the childs trust has been obtained.

c. Allow the child to keep a security object such as a toy or blanket during the examination. The best place to examine the toddler is on the parents lap. Toddlers understand symbols; therefore, a security object is helpful. Initially, the focus is more on the parent, which allows the child to adjust gradually and to become familiar with you. A 2-year-old child does not like to take off his or her clothes. Therefore, ask the parent to undress one body part at a time.

32. A 30-year-old female patient is describing feelings of hopelessness and depression. She has attempted selfmutilation and has a history of suicide attempts. She describes difficulty sleeping at night and has lost 10 pounds in the past month. Which of these statements or questions is the nurses best response in this situation? a. Do you have a weapon? b. How do other people treat you? c. Are you feeling so hopeless that you feel like hurting yourself now? d. People often feel hopeless, but the feelings resolve within a few weeks.

c. Are you feeling so hopeless that you feel like hurting yourself now? When the person expresses feelings of hopelessness, despair, or grief, assessing the risk of physical harm to him or herself is important. This process begins with more general questions. If the answers are affirmative, then the assessment continues with more specific questions.

3. The nurse is aware that intimate partner violence (IPV) screening should occur with which situation? a. When IPV is suspected b. When a woman has an unexplained injury c. As a routine part of each health care encounter d. When a history of abuse in the family is known

c. As a routine part of each health care encounter Many nursing professional organizations have called for routine, universal screening for IPV to assist women in getting help for the problem.

13. The nurse is examining a 3-year-old child who was brought to the emergency department after a fall. Which bruise, if found, would be of most concern? a. Bruise on the knee b. Bruise on the elbow c. Bruising on the abdomen d. Bruise on the shin

c. Bruising on the abdomen Studies have shown that children who are walking often have bruises over the bony prominences of the front of their bodies. Other studies have found that bruising in atypical places such as the buttocks, hands, feet, and abdomen were exceedingly rare and should arouse concern.

21. A patient repeatedly seems to have difficulty coming up with a word. He says, I was on my way to work, and when I got there, the thing that you step into that goes up in the air was so full that I decided to take the stairs. The nurse will note on his chart that he is using or experiencing: a. Blocking b. Neologism c. Circumlocution d. Circumstantiality

c. Circumlocution Circumlocution is a roundabout expression, substituting a phrase when one cannot think of the name of the object.

7. The nurse should measure rectal temperatures in which of these patients? a. School-age child b. Older adult c. Comatose adult d. Patient receiving oxygen by nasal cannula

c. Comatose adult Rectal temperatures should be taken when the other routes are impractical, such as for comatose or confused persons, for those in shock, or for those who cannot close the mouth because of breathing or oxygen tubes, a wired mandible, or other facial dysfunctions.

31. The nurse is taking an initial blood pressure reading on a 72-year-old patient with documented hypertension. How should the nurse proceed? a. Cuff should be placed on the patients arm and inflated 30 mm Hg above the patients pulse rate. b. Cuff should be inflated to 200 mm Hg in an attempt to obtain the most accurate systolic reading. c. Cuff should be inflated 30 mm Hg above the point at which the palpated pulse disappears. d. After confirming the patients previous blood pressure readings, the cuff should be inflated 30 mm Hg above the highest systolic reading recorded.

c. Cuff should be inflated 30 mm Hg above the point at which the palpated pulse disappears. An auscultatory gap occurs in approximately 5% of the people, most often in those with hypertension. To check for the presence of an auscultatory gap, the cuff should be inflated 20 to 30 mm Hg beyond the point at which the palpated pulse disappears.

3. During a seminar on cultural aspects of nursing, the nurse recognizes that the definition stating the specific and distinct knowledge, beliefs, skills, and customs acquired by members of a society reflects which term? a. Mores b. Norms c. Culture d. Social learning

c. Culture The culture that develops in any given society is always specific and distinctive, encompassing all of the knowledge, beliefs, customs, and skills acquired by members of the society. The other terms do not fit the given definition.

50. During an examination, the nurse notices that a female patient has a round moon face, central trunk obesity, and a cervical hump. Her skin is fragile with bruises. The nurse determines that the patient has which condition? a. Marfan syndrome b. Gigantism c. Cushing syndrome d. Acromegaly

c. Cushing syndrome Cushing syndrome is characterized by weight gain and edema with central trunk and cervical obesity (buffalo hump) and round plethoric face (moon face). Excessive catabolism causes muscle wasting; weakness; thin arms and legs; reduced height; and thin, fragile skin with purple abdominal striae, bruising, and acne. (See Table 9-5, Abnormalities in Body Height and Proportion, for the definitions of the other conditions.)

24. Which of these interventions is most appropriate when the nurse is planning nutritional interventions for a healthy, active 74-year-old woman? a. Decreasing the amount of carbohydrates to prevent lean muscle catabolism b. Increasing the amount of soy and tofu in her diet to promote bone growth and reverse osteoporosis c. Decreasing the number of calories she is eating because of the decrease in energy requirements from the loss of lean body mass d. Increasing the number of calories she is eating because of the increased energy needs of the older adult

c. Decreasing the number of calories she is eating because of the decrease in energy requirements Important nutritional features of the older years are a decrease in energy requirements as a result of loss of lean body mass, the most metabolically active tissue, and an increase in fat mass.

7. The nurse is preparing to percuss the abdomen of a patient. The purpose of the percussion is to assess the __________ of the underlying tissue. a. Turgor b. Texture c. Density d. Consistency

c. Density Percussion yields a sound that depicts the location, size, and density of the underlying organ. Turgor and texture are assessed with palpation.

12. When measuring a patients body temperature, the nurse keeps in mind that body temperature is influenced by: a. Constipation. b. Patients emotional state. c. Diurnal cycle. d. Nocturnal cycle.

c. Diurnal cycle. Normal temperature is influenced by the diurnal cycle, exercise, and age. The other responses do not influence body temperature.

9. After a class on culture and ethnicity, the new graduate nurse reflects a correct understanding of the concept of ethnicity with which statement? a. Ethnicity is dynamic and ever changing. b. Ethnicity is the belief in a higher power. c. Ethnicity pertains to a social group within the social system that claims shared values and traditions. d. Ethnicity is learned from birth through the processes of language acquisition and socialization.

c. Ethnicity pertains to a social group within the social system that claims shared values and traditions. Ethnicity pertains to a social group within the social system that claims to have variable traits, such as a common geographic origin, migratory status, religion, race, language, values, traditions, symbols, or food preferences.Culture is dynamic, ever changing, and learned from birth through the processes of language acquisition and socialization. Religion is the belief in a higher power.

21. When the nurse asks for a description of who lives with a child, the method of discipline, and the support system of the child, what part of the assessment is being performed? a. Family history b. Review of systems c. Functional assessment d. Reason for seeking care

c. Functional assessment Functional assessment includes interpersonal relationships and home environment. Family history includes illnesses in family members; a review of systems includes questions about the various body systems; and the reason for seeking care is the rationale for requesting health care.

20. An older Mexican-American woman with traditional beliefs has been admitted to an inpatient care unit. A culturally sensitive nurse would: a. Contact the hospital administrator about the best course of action. b. Automatically get a curandero for her, because requesting one herself is not culturally appropriate. c. Further assess the patients cultural beliefs and offer the patient assistance in contacting a curandero or priest if she desires. d. Ask the family what they would like to do because Mexican-Americans traditionally give control of decision making to their families.

c. Further assess the patients cultural beliefs and offer the patient assistance in contacting a curandero or priest if she desires. In addition to seeking help from the biomedical or scientific health care provider, patients may also seek help from folk or religious healers. Some people, such as those of Mexican-American or American-Indian origins, may believe that the cure is incomplete unless the body, mind, and spirit are also healed (although the division of the person into parts is a Western concept).

12. The nurse is planning to assess new memory with a patient. The best way for the nurse to do this would be to: a. Administer the FACT test. b. Ask him to describe his first job. c. Give him the Four Unrelated Words Test. d. Ask him to describe what television show he was watching before coming to the clinic.

c. Give him the Four Unrelated Words Test. Ask questions that can be corroborated, which screens for the occasional person who confabulates or makes up answers to fill in the gaps of memory loss. The Four Unrelated Words Test tests the persons ability to lay down new memories and is a highly sensitive and valid memory test

25. A 16-year-old girl is being seen at the clinic for gastrointestinal complaints and weight loss. The nurse determines that many of her complaints may be related to erratic eating patterns, eating predominantly fast foods, and high caffeine intake. In this situation, which is most appropriate when collecting current dietary intake information? a. Scheduling a time for direct observation of the adolescent during meals b. Asking the patient for a 24-hour diet recall, and assuming it to be reflective of a typical day for her c. Having the patient complete a food diary for 3 days, including 2 weekdays and 1 weekend day d. Using the food frequency questionnaire to identify the amount of intake of specific foods

c. Having the patient complete a food diary for 3 days, including 2 weekdays and 1 weekend day Food diaries require the individual to write down everything consumed for a certain time period. Because of the erratic eating patterns of this individual, assessing dietary intake over a few days would produce more accurate information regarding eating patterns. Direct observation is best used with young children or older adults.

34. A patient is seen in the clinic for complaints of fainting episodes that started last week. How should the nurse proceed with the examination? a. Blood pressure readings are taken in both the arms and the thighs. b. The patient is assisted to a lying position, and his blood pressure is taken. c. His blood pressure is recorded in the lying, sitting, and standing positions. d. His blood pressure is recorded in the lying and sitting positions; these numbers are then averaged to obtain a mean blood pressure.

c. His blood pressure is recorded in the lying, sitting, and standing positions. If the person is known to have hypertension, is taking antihypertensive medications, or reports a history of fainting or syncope, then the blood pressure reading should be taken in three positions: lying, sitting, and standing.

27. A patient states, I feel so sad all of the time. I cant feel happy even doing things I used to like to do. He also states that he is tired, sleeps poorly, and has no energy. To differentiate between a dysthymic disorder and a major depressive disorder, the nurse should ask which question? a. Have you had any weight changes? b. Are you having any thoughts of suicide? c. How long have you been feeling this way? d. Are you having feelings of worthlessness?

c. How long have you been feeling this way? Major depressive disorder is characterized by one or more major depressive episodes, that is, at least 2 weeks of depressed mood or loss of interest accompanied by at least four additional symptoms of depression. Dysthymic disorder is characterized by at least 2 years of depressed mood for more days than not, accompanied by additional depressive symptoms.

7. The nurse is conducting a patient interview. Which statement made by the patient should the nurse more fully explore during the interview? a. I sleep like a baby. b. I have no health problems. c. I never did too good in school. d. I am not currently taking any medications

c. I never did too good in school. In every mental status examination, the following factors from the health history that could affect the findings should be noted: any known illnesses or health problems, such as alcoholism or chronic renal disease; current medications, the side effects of which may cause confusion or depression; the usual educational and behavioral level, noting this level as the patients normal baseline and not expecting a level of performance on the mental status examination to exceed it; and responses to personal history questions, indicating current stress, social interaction patterns, and sleep habits.

36. The nurse is helping another nurse to take a blood pressure reading on a patients thigh. Which action is correct regarding thigh pressure? a. Either the popliteal or femoral vessels should be auscultated to obtain a thigh pressure. b. The best position to measure thigh pressure is the supine position with the knee slightly bent. c. If the blood pressure in the arm is high in an adolescent, then it should be compared with the thigh pressure. d. The thigh pressure is lower than the pressure in the arm, which is attributable to the distance away from the heart and the size of the popliteal vessels.

c. If the blood pressure in the arm is high in an adolescent, then it should be compared with the thigh pressure. When blood pressure measured at the arm is excessively high, particularly in adolescents and young adults, it is compared with thigh pressure to check for coarctation of the aorta. The popliteal artery is auscultated for the reading. Generally, thigh pressure is higher than that of the arm; however, if coarctation of the artery is present, then arm pressures are higher than thigh pressures.

10. The nurse is unable to identify any changes in sound when percussing over the abdomen of an obese patient. What should the nurse do next? a. Ask the patient to take deep breaths to relax the abdominal musculature. b. Consider this finding as normal, and proceed with the abdominal assessment. c. Increase the amount of strength used when attempting to percuss over the abdomen. d. Decrease the amount of strength used when attempting to percuss over the abdomen.

c. Increase the amount of strength used when attempting to percuss over the abdomen. The thickness of the persons body wall will be a factor. The nurse needs a stronger percussion stroke for persons with obese or very muscular body walls. The force of the blow determines the loudness of the note. The other actions are not correct.

4. A patient states that the pain medication is not working and rates his postoperative pain at a 10 on a 1-to-10 scale. Which of these assessment findings indicates an acute pain response to poorly controlled pain? a. Confusion b. Hyperventilation c. Increased blood pressure and pulse d. Depression

c. Increased blood pressure and pulse Responses to poorly controlled acute pain include tachycardia, elevated blood pressure, and hypoventilation. Confusion and depression are associated with poorly controlled chronic pain (see Table 10-1).

22. When assessing the force, or strength, of a pulse, the nurse recalls that the pulse: a. Is usually recorded on a 0- to 2-point scale. b. Demonstrates elasticity of the vessel wall. c. Is a reflection of the hearts stroke volume. d. Reflects the blood volume in the arteries during diastole.

c. Is a reflection of the hearts stroke volume. The heart pumps an amount of blood (the stroke volume) into the aorta. The force flares the arterial walls and generates a pressure wave, which is felt in the periphery as the pulse.

18. The nurse is performing the Denver II screening test on a 12-month-old infant during a routine well-child visit. The nurse should tell the infants parents that the Denver II: a. Tests three areas of development: cognitive, physical, and psychological b. Will indicate whether the child has a speech disorder so that treatment can begin. c. Is a screening instrument designed to detect children who are slow in development. d. Is a test to determine intellectual ability and may indicate whether problems will develop later in school.

c. Is a screening instrument designed to detect children who are slow in development. The Denver II is a screening instrument designed to detect developmental delays in infants and preschoolers. It tests four functions: gross motor, language, fine motor-adaptive, and personal-social. The Denver II is not an intelligence test; it does not predict current or future intellectual ability. It is not diagnostic; it does not suggest treatment regimens.

12. A 17-year-old single mother is describing how difficult it is to raise a 3-year-old child by herself. During the course of the interview she states, I cant believe my boyfriend left me to do this by myself! What a terrible thing to do to me! Which of these responses by the nurse uses empathy? a. You feel alone. b. You cant believe he left you alone? c. It must be so hard to face this all alone. d. I would be angry, too; raising a child alone is no picnic.

c. It must be so hard to face this all alone. An empathetic response recognizes the feeling and puts it into words. It names the feeling, allows its expression, and strengthens rapport. Other empathetic responses are, This must be very hard for you, I understand, or simply placing your hand on the persons arm. Simply reflecting the persons words or agreeing with the person is not an empathetic response.

15. During an interview, a woman says, I have decided that I can no longer allow my children to live with their fathers violence, but I just cant seem to leave him. Using interpretation, the nurses best response would be: a. You are going to leave him? b. If you are afraid for your children, then why cant you leave? c. It sounds as if you might be afraid of how your husband will respond. d. It sounds as though you have made your decision. I think it is a good one.

c. It sounds as if you might be afraid of how your husband will respond. This statement is not based on ones inference or conclusion. It links events, makes associations, or implies cause. Interpretation also ascribes feelings and helps the person understand his or her own feelings in relation to the verbal message. The other statements do not reflect interpretation.

1. When evaluating a patients pain, the nurse knows that an example of acute pain would be: a. Arthritic pain. b. Fibromyalgia. c. Kidney stones. d. Low back pain.

c. Kidney stones. Acute pain is short-term and dissipates after an injury heals, such as with kidney stones. The other conditions are examples of chronic pain during which the pain continues for 6 months or longer and does not stop when the injury heals.

5. Which term refers to a wound produced by the tearing or splitting of body tissue, usually from blunt impact over a bony surface? a. Abrasion b. Contusion c. Laceration d. Hematoma

c. Laceration The term laceration refers to a wound produced by the tearing or splitting of body tissue. An abrasion is caused by the rubbing of the skin or mucous membrane. A contusion is injury to tissues without breakage of skin, and a hematoma is a localized collection of extravasated blood.

6. The nurse is assessing a 30-year-old unemployed immigrant from an underdeveloped country who has been in the United States for 1 month. Which of these problems related to his nutritional status might the nurse expect to find? a. Obesity b. Hypotension c. Osteomalacia (softening of the bones) d. Coronary artery disease

c. Osteomalacia (softening of the bones) General undernutrition, hypertension, diarrhea, lactose intolerance, osteomalacia, scurvy, and dental caries are among the more common nutrition-related problems of new immigrants from developing countries.

44. When considering the concepts related to blood pressure, the nurse knows that the concept of mean arterial pressure (MAP) is best described by which statement? a. MAP is the pressure of the arterial pulse. b. MAP reflects the stroke volume of the heart. c. MAP is the pressure forcing blood into the tissues, averaged over the cardiac cycle. d. MAP is an average of the systolic and diastolic blood pressures and reflects tissue perfusion

c. MAP is the pressure forcing blood into the tissues, averaged over the cardiac cycle. MAP is the pressure that forces blood into the tissues, averaged over the cardiac cycle. Stroke volume is reflected by the blood pressure. MAP is not an arithmetic average of systolic and diastolic pressures because diastole lasts longer; rather, it is a value closer to diastolic pressure plus one third of the pulse pressure.

2. Receiving is a part of the communication process. Which receiver is most likely to misinterpret a message sent by a health care professional? a. Well-adjusted adolescent who came in for a sports physical b. Recovering alcoholic who came in for a basic physical examination c. Man whose wife has just been diagnosed with lung cancer d. Man with a hearing impairment who uses sign language to communicate and who has an interpreter with him

c. Man whose wife has just been diagnosed with lung cancer The receiver attaches meaning determined by his or her experiences, culture, self-concept, and current physical and emotional states. The man whose wife has just been diagnosed with lung cancer may be experiencing emotions that affect his receiving

27. Which of these conditions is due to an inadequate intake of both protein and calories? a. Obesity b. Bulimia c. Marasmus d. Kwashiorkor

c. Marasmus Marasmus, protein-calorie malnutrition, is due to an inadequate intake of protein and calories or prolonged starvation. Obesity is due to caloric excess; bulimia is an eating disorder. Kwashiorkor is protein malnutrition.

4. During a session on substance abuse, the nurse is reviewing statistics with the class. For persons aged 12 years and older, which illicit substance was most commonly used? a. Crack cocaine b. Heroin c. Marijuana d. Hallucinogens

c. Marijuana In persons age 12 years and older who reported using during the past month, marijuana (hashish) was the most commonly used illicit drug reported.

19. If an American Indian woman has come to the clinic to seek help with regulating her diabetes, then the nurse can expect that she: a. Will comply with the treatment prescribed. b. Has obviously given up her belief in naturalistic causes of disease. c. May also be seeking the assistance of a shaman or medicine man. d. Will need extra help in dealing with her illness and may be experiencing a crisis of faith.

c. May also be seeking the assistance of a shaman or medicine man. When self-treatment is unsuccessful, the individual may turn to the lay or folk healing systems, to spiritual or religious healing, or to scientific biomedicine. In addition to seeking help from a biomedical or scientific health care provider, patients may also seek help from folk or religious healers.

3. The nurse is assessing a 75-year-old man. As the nurse begins the mental status portion of the assessment, the nurse expects that this patient: a. Will have no decrease in any of his abilities, including response time. b. Will have difficulty on tests of remote memory because this ability typically decreases with age. c. May take a little longer to respond, but his general knowledge and abilities should not have declined. d. Will exhibit had a decrease in his response time because of the loss of language and a decrease in general knowledge.

c. May take a little longer to respond, but his general knowledge and abilities should not have declined. The aging process leaves the parameters of mental status mostly intact. General knowledge does not decrease, and little or no loss in vocabulary occurs. Response time is slower than in a youth. It takes a little longer for the brain to process information and to react to it. Recent memory, which requires some processing, is somewhat decreased with aging, but remote memory is not affected.

8. The nurse is preparing to measure the length, weight, chest, and head circumference of a 6-month-old infant. Which measurement technique is correct? a. Measuring the infants length by using a tape measure b. Weighing the infant by placing him or her on an electronic standing scale c. Measuring the chest circumference at the nipple line with a tape measure d. Measuring the head circumference by wrapping the tape measure over the nose and cheekbones

c. Measuring the chest circumference at the nipple line with a tape measure To measure the chest circumference, the tape is encircled around the chest at the nipple line. The length should be measured on a horizontal measuring board. Weight should be measured on a platform-type balance scale. Head circumference is measured with the tape around the head, aligned at the eyebrows, and at the prominent frontal and occipital bonesthe widest span is correct.

9. The nurse is taking a family history. Important diseases or problems about which the patient should be specifically asked include: a. Emphysema. b. Head trauma. c. Mental illness. d. Fractured bones.

c. Mental illness. Questions concerning any family history of heart disease, high blood pressure, stroke, diabetes, obesity, blood disorders, breast and ovarian cancers, colon cancer, sickle cell anemia, arthritis, allergies, alcohol or drug addiction, mental illness, suicide, seizure disorder, kidney disease, and tuberculosis should be asked.

29. The nurse is performing a mental status examination. Which statement is true regarding the assessment of mental status? a. Mental status assessment diagnoses specific psychiatric disorders. b. Mental disorders occur in response to everyday life stressors. c. Mental status functioning is inferred through the assessment of an individuals behaviors. d. Mental status can be directly assessed, similar to other systems of the body (e.g., heart sounds, breath sounds).

c. Mental status functioning is inferred through the assessment of an individuals behaviors. Mental status functioning is inferred through the assessment of an individuals behaviors. It cannot be directly assessed like the characteristics of the skin or heart sounds.

During an examination, the nurse can assess mental status by which activity? a. Examining the patients electroencephalogram b. Observing the patient as he or she performs an intelligence quotient (IQ) test c. Observing the patient and inferring health or dysfunction d. Examining the patients response to a specific set of questions

c. Observing the patient and inferring health or dysfunction Mental status cannot be directly scrutinized like the characteristics of skin or heart sounds. Its functioning is inferred through an assessment of an individuals behaviors, such as consciousness, language, mood and affect, and other aspects.

40. In a patient with acromegaly, the nurse will expect to discover which assessment findings? a. Heavy, flattened facial features b. Growth retardation and a delayed onset of puberty c. Overgrowth of bone in the face, head, hands, and feet d. Increased height and weight and delayed sexual development

c. Overgrowth of bone in the face, head, hands, and feet Excessive secretions of growth hormone in adulthood after normal completion of body growth causes an overgrowth of the bones in the face, head, hands, and feet but no change in height.

2. Which statement indicates that the nurse understands the pain experienced by an older adult? a. Older adults must learn to tolerate pain. b. Pain is a normal process of aging and is to be expected. c. Pain indicates a pathologic condition or an injury and is not a normal process of aging. d. Older individuals perceive pain to a lesser degree than do younger individuals.

c. Pain indicates a pathologic condition or an injury and is not a normal process of aging. Pain indicates a pathologic condition or an injury and should never be considered something that an older adult should expect or tolerate. Pain is not a normal process of aging, and no evidence suggests that pain perception is reduced with aging

11. Which of these statements represents subjective data the nurse obtained from the patient regarding the patients skin? a. Skin appears dry. b. No lesions are obvious. c. Patient denies any color change. d. Lesion is noted on the lateral aspect of the right arm.

c. Patient denies any color change. The history should be limited to patient statements or subjective datafactors that the person says were or were not present

10. The nurse is comparing the concepts of religion and spirituality. Which of the following is an appropriate component of ones spirituality? a. Belief in and the worship of God or gods b. Attendance at a specific church or place of worship c. Personal effort made to find purpose and meaning in life d. Being closely tied to ones ethnic background

c. Personal effort made to find purpose and meaning in life Spirituality refers to each persons unique life experiences and his or her personal effort to find purpose and meaning in life. The other responses apply to religion.

31. The nurse is planning health teaching for a 65-year-old woman who has had a cerebrovascular accident (stroke) and has aphasia. Which of these questions is most important to use when assessing mental status in this patient? a. Please count backward from 100 by seven. b. I will name three items and ask you to repeat them in a few minutes. c. Please point to articles in the room and parts of the body as I name them. d. What would you do if you found a stamped, addressed envelope on the sidewalk?

c. Please point to articles in the room and parts of the body as I name them. Additional tests for persons with aphasia include word comprehension (asking the individual to point to articles in the room or parts of the body), reading (asking the person to read available print), and writing (asking the person to make up and write a sentence).

6. The nurse is assessing an 80-year-old male patient. Which assessment findings would be considered normal? a. Increase in body weight from his younger years b. Additional deposits of fat on the thighs and lower legs c. Presence of kyphosis and flexion in the knees and hips d. Change in overall body proportion, including a longer trunk and shorter extremities

c. Presence of kyphosis and flexion in the knees and hips Changes that occur in the aging person include more prominent bony landmarks, decreased body weight (especially in men), a decrease in subcutaneous fat from the face and periphery, and additional fat deposited on the abdomen and hips. Postural changes of kyphosis and slight flexion in the knees and hips also occur.

20. A 50-year-old woman with elevated total cholesterol and triglyceride levels is visiting the clinic to find out about her laboratory results. What would be important for the nurse to include in patient teaching in relation to these tests? a. The risks of undernutrition should be included. b. Offer methods to reduce the stress in her life. c. Provide information regarding a diet low in saturated fat. d. This condition is hereditary; she can do nothing to change the levels.

c. Provide information regarding a diet low in saturated fat. The patient with elevated cholesterol and triglyceride levels should be taught about eating a healthy diet that limits the intake of foods high in saturated fats or trans fats. Reducing dietary fats is part of the treatment for this condition. The other responses are not pertinent to her condition.

23. The nurse is performing a review of systems on a 76-year-old patient. Which of these statements is correct for this situation? a. The questions asked are identical for all ages. b. The interviewer will start incorporating different questions for patients 70 years of age and older. c. Questions that are reflective of the normal effects of aging are added. d. At this age, a review of systems is not necessarythe focus should be on current problems.

c. Questions that are reflective of the normal effects of aging are added. The health history includes the same format as that described for the younger adult, as well as some additional questions. These additional questions address ways in which the activities of daily living may have been affected by the normal aging processes or by the effects of chronic illness or disability.

Words Test, the nurse would be concerned if she could not ____ four unrelated words ____. a. Invent; within 5 minutes b. Invent; within 30 seconds c. Recall; after a 30-minute delay d. Recall; after a 60-minute delay

c. Recall; after a 30-minute delay The Four Unrelated Words Test tests the persons ability to lay down new memories. It is a highly sensitive and valid memory test. It requires more effort than the recall of personal or historic events. To the person say, I am going to say four words. I want you to remember them. In a few minutes I will ask you to recall them. After 5 minutes, ask for the four words. The normal response for persons under 60 years is an accurate three- or fourword recall after a 5-, 10-, and 30-minute delay.

31. During a class on cultural practices, the nurse hears the term cultural taboo. Which statement illustrates the concept of a cultural taboo? a. Believing that illness is a punishment of sin b. Trying prayer before seeking medical help c. Refusing to accept blood products as part of treatment d. Stating that a childs birth defect is the result of the parents sins

c. Refusing to accept blood products as part of treatment Cultural taboos are practices that are to be avoided, such as receiving blood products, eating pork, and consuming caffeine. The other answers do not reflect cultural taboos.

37. During an interview, the nurse would expect that most of the interview will take place at what distance? a. Intimate zone b. Personal distance c. Social distance d. Public distance

c. Social distance Social distance, 4 to 12 feet, is usually the distance category for most of the interview. Public distance, over 12 feet, is too much distance; the intimate zone is inappropriate, and the personal distance will be used for the physical assessment.

36. Which of these statements is true regarding the use of Standard Precautions in the health care setting? a. Standard Precautions apply to all body fluids, including sweat. b. Use alcohol-based hand rub if hands are visibly dirty. c. Standard Precautions are intended for use with all patients, regardless of their risk or presumed infection status. d. Standard Precautions are to be used only when nonintact skin, excretions containing visible blood, or expected contact with mucous membranes is present.

c. Standard Precautions are intended for use with all patients, regardless of their risk or presumed infection status. Standard Precautions are designed to reduce the risk of transmission of microorganisms from both recognized and unrecognized sources and are intended for use for all patients, regardless of their risk or presumed infection status. Standard Precautions apply to blood and all other body fluids, secretions and excretions except sweatregardless of whether they contain visible blood, nonintact skin, or mucous membranes. Hands should be washed with soap and water if visibly soiled with blood or body fluids. Alcohol-based hand rubs can be used if hands are not visibly soiled.

43. What type of blood pressure measurement error is most likely to occur if the nurse does not check for the presence of an auscultatory gap? a. Diastolic blood pressure may not be heard. b. Diastolic blood pressure may be falsely low. c. Systolic blood pressure may be falsely low. d. Systolic blood pressure may be falsely high.

c. Systolic blood pressure may be falsely low. If an auscultatory gap is undetected, then a falsely low systolic or falsely high diastolic reading may result, which is common in patients with hypertension.

22. During an examination, the nurse notes that a patient is exhibiting flight of ideas. Which statement by the patient is an example of flight of ideas? a. My stomach hurts. Hurts, spurts, burts. b. Kiss, wood, reading, ducks, onto, maybe. c. Take this pill? The pill is red. I see red. Red velvet is soft, soft as a babys bottom. d. I wash my hands, wash them, wash them. I usually go to the sink and wash my hands.

c. Take this pill? The pill is red. I see red. Red velvet is soft, soft as a babys bottom. Flight of ideas is demonstrated by an abrupt change, rapid skipping from topic to topic, and practically continuous flow of accelerated speech. Topics usually have recognizable associations or are plays on words.

41. The nurse is preparing to examine a 6-year-old child. Which action is most appropriate? a. The thorax, abdomen, and genitalia are examined before the head. b. Talking about the equipment being used is avoided because doing so may increase the childs anxiety. c. The nurse should keep in mind that a child at this age will have a sense of modesty. d. The child is asked to undress from the waist up.

c. The nurse should keep in mind that a child at this age will have a sense of modesty. A 6-year-old child has a sense of modesty. The child should undress him or herself, leaving underpants on and using a gown or drape. A school-age child is curious to know how equipment works, and the sequence should progress from the childs head to the toes.

17. The nurse is taking temperatures in a clinic with a TMT. Which statement is true regarding use of the TMT? a. A tympanic temperature is more time consuming than a rectal temperature. b. The tympanic method is more invasive and uncomfortable than the oral method. c. The risk of cross-contamination is reduced, compared with the rectal route. d. The tympanic membrane most accurately reflects the temperature in the ophthalmic artery.

c. The risk of cross-contamination is reduced, compared with the rectal route. The TMT is a noninvasive, nontraumatic device that is extremely quick and efficient. The chance of crosscontamination with the TMT is minimal because the ear canal is lined with skin, not mucous membranes.

25. A patients blood pressure is 118/82 mm Hg. He asks the nurse, What do the numbers mean? The nurses best reply is: a. The numbers are within the normal range and are nothing to worry about. b. The bottom number is the diastolic pressure and reflects the stroke volume of the heart. c. The top number is the systolic blood pressure and reflects the pressure of the blood against the arteries when the heart contracts. d. The concept of blood pressure is difficult to understand. The primary thing to be concerned about is the top number, or the systolic blood pressure.

c. The top number is the systolic blood pressure and reflects the pressure of the blood against the The systolic pressure is the maximum pressure felt on the artery during left ventricular contraction, or systole. The diastolic pressure is the elastic recoil, or resting, pressure that the blood constantly exerts in between each contraction. The nurse should answer the patients question and use terms he can understand.

33. The nurse is providing instructions to newly hired graduates for the minimental state examination (MMSE). Which statement best describes this examination? a. Scores below 30 indicate cognitive impairment. b. The MMSE is a good tool to evaluate mood and thought processes. c. This examination is a good tool to detect delirium and dementia and to differentiate these from psychiatric mental illness. d. The MMSE is useful tool for an initial evaluation of mental status. Additional tools are needed to evaluate cognition changes over time.

c. This examination is a good tool to detect delirium and dementia and to differentiate these from psychiatric mental illness. The MMSE is a quick, easy test of 11 questions and is used for initial and serial evaluations and can demonstrate a worsening or an improvement of cognition over time and with treatment. It evaluates cognitive functioning, not mood or thought processes. MMSE is a good screening tool to detect dementia and delirium and to differentiate these from psychiatric mental illness.

32. The nurse makes this comment to a patient, I know it may be hard, but you should do what the doctor ordered because she is the expert in this field. Which statement is correct about the nurses comment? a. This comment is inappropriate because it shows the nurses bias. b. This comment is appropriate because members of the health care team are experts in their area of patient care. c. This type of comment promotes dependency and inferiority on the part of the patient and is best avoided in an interview situation. d. Using authority statements when dealing with patients, especially when they are undecided about an issue, is necessary at times.

c. This type of comment promotes dependency and inferiority on the part of the patient and is best avoided in an interview situation. Using authority responses promotes dependency and inferiority. Avoiding the use of authority is best. Although the health care provider and patient do not have equal professional knowledge, both have equally worthy roles in the health process. The other statements are not correct.

14. A 60-year-old male patient has been treated for pneumonia for the past 6 weeks. He is seen today in the clinic for an unexplained weight loss of 10 pounds over the last 6 weeks. The nurse knows that: a. Weight loss is probably the result of unhealthy eating habits. b. Chronic diseases such as hypertension cause weight loss. c. Unexplained weight loss often accompanies short-term illnesses. d. Weight loss is probably the result of a mental health dysfunction.

c. Unexplained weight loss often accompanies short-term illnesses. An unexplained weight loss may be a sign of a short-term illness or a chronic illness such as endocrine disease, malignancy, depression, anorexia nervosa, or bulimia.

19. The nurse is unable to palpate the right radial pulse on a patient. The best action would be to: a. Auscultate over the area with a fetoscope. b. Use a goniometer to measure the pulsations. c. Use a Doppler device to check for pulsations over the area. d. Check for the presence of pulsations with a stethoscope.

c. Use a Doppler device to check for pulsations over the area. Doppler devices are used to augment pulse or blood pressure measurements. Goniometers measure joint range of motion. A fetoscope is used to auscultate fetal heart tones. Stethoscopes are used to auscultate breath, bowel, and heart sounds.

19. A nurse is taking complete health histories on all of the patients attending a wellness workshop. On the history form, one of the written questions asks, You dont smoke, drink, or take drugs, do you? This question is an example of: a. Talking too much. b. Using confrontation. c. Using biased or leading questions. d. Using blunt language to deal with distasteful topics

c. Using biased or leading questions. This question is an example of using leading or biased questions. Asking, You dont smoke, do you? implies that one answer is better than another. If the person wants to please someone, then he or she is either forced to answer in a way that corresponds to his or her implied values or is made to feel guilty when admitting the other answer.

29. A 50-year-old patient has been brought to the emergency department after a housemate found that the patient could not get out of bed alone. He has lived in a group home for years but for several months has not participated in the activities and has stayed in his room. The nurse assesses for signs of undernutrition, and an x-ray study reveals that he has osteomalacia, which is a deficiency of: a. Iron. b. Riboflavin. c. Vitamin D and calcium. d. Vitamin C.

c. Vitamin D and calcium. Osteomalacia results from a deficiency of vitamin D and calcium in adults. Iron deficiency would result in anemia, riboflavin deficiency would result in magenta tongue, and vitamin C deficiency would result in scurvy.

35. The most important step that the nurse can take to prevent the transmission of microorganisms in the hospital setting is to: a. Wear protective eye wear at all times. b. Wear gloves during any and all contact with patients. c. Wash hands before and after contact with each patient. d. Clean the stethoscope with an alcohol swab between patients.

c. Wash hands before and after contact with each patient. The most important step to decrease the risk of microorganism transmission is to wash hands promptly and thoroughly before and after physical contact with each patient. Stethoscopes should also be cleansed with an alcohol swab before and after each patient contact. The best routine is to combine stethoscope rubbing with hand hygiene each time hand hygiene is performed.

23. The nurse is examining a patients lower leg and notices a draining ulceration. Which of these actions is most appropriate in this situation? a. Washing hands, and contacting the physician b. Continuing to examine the ulceration, and then washing hands c. Washing hands, putting on gloves, and continuing with the examination of the ulceration d. Washing hands, proceeding with rest of the physical examination, and then continuing with the examination of the leg ulceration

c. Washing hands, putting on gloves, and continuing with the examination of the ulceration The examiner should wear gloves when the potential contact with any body fluids is present. In this situation, the nurse should wash his or her hands, put on gloves, and continue examining the ulceration.

14. In response to a question about stress, a 39-year-old woman tells the nurse that her husband and mother both died in the past year. Which response by the nurse is most appropriate? a. This has been a difficult year for you. b. I dont know how anyone could handle that much stress in 1 year! c. What did you do to cope with the loss of both your husband and mother? d. That is a lot of stress; now lets go on to the next section of your history.

c. What did you do to cope with the loss of both your husband and mother? Questions about coping and stress management include questions regarding the kinds of stresses in ones life, especially in the last year, any changes in lifestyle or any current stress, methods tried to relieve stress, and whether these methods have been helpful.

8. The nurse is asking an adolescent about illicit substance abuse. The adolescent answers, Yes, Ive used marijuana at parties with my friends. What is the next question the nurse should ask? a. Who are these friends? b. Do your parents know about this? c. When was the last time you used marijuana? d. Is this a regular habit?

c. When was the last time you used marijuana? If a patient admits to the use of illicit substances, then the nurse should ask, When was the last time you used drugs? and How much did you take that time? The other questions may be considered accusatory and are not conducive to gathering information.

10. A patient has been in the intensive care unit for 10 days. He has just been moved to the medical-surgical unit, and the admitting nurse is planning to perform a mental status examination. During the tests of cognitive function, the nurse would expect that he: a. May display some disruption in thought content. b. Will state, I am so relieved to be out of intensive care. c. Will be oriented to place and person, but the patient may not be certain of the date. d. May show evidence of some clouding of his level of consciousness.

c. Will be oriented to place and person, but the patient may not be certain of the date. The nurse can discern the orientation of cognitive function through the course of the interview or can directly and tactfully ask, Some people have trouble keeping up with the dates while in the hospital. Do you know todays date? Many hospitalized people have trouble with the exact date but are fully oriented on the remaining items.

24. During the examination, offering some brief teaching about the patients body or the examiners findings is often appropriate. Which one of these statements by the nurse is most appropriate? a. Your atrial dysrhythmias are under control. b. You have pitting edema and mild varicosities. c. Your pulse is 80 beats per minute, which is within the normal range. d. Im using my stethoscope to listen for any crackles, wheezes, or rubs.

c. Your pulse is 80 beats per minute, which is within the normal range. The sharing of some information builds rapport, as long as the patient is able to understand the terminology.

38. When examining an infant, the nurse should examine which area first? a. Ear b. Nose c. Throat d. Abdomen

d. Abdomen The least-distressing steps are performed first, saving the invasive steps of the examination of the eye, ear, nose, and throat until last.

13. A patient is complaining of severe knee pain after twisting it during a basketball game and is requesting pain medication. Which action by the nurse is appropriate? a. Completing the physical examination first and then giving the pain medication b. Telling the patient that the pain medication must wait until after the x-ray images are completed c. Evaluating the full range of motion of the knee and then medicating for pain d. Administering pain medication and then proceeding with the assessment

d. Administering pain medication and then proceeding with the assessment According to the American Pain Society (1992), In cases in which the cause of acute pain is uncertain, establishing a diagnosis is a priority, but symptomatic treatment of pain should be given while the investigation is proceeding. With occasional exceptions, (e.g., the initial examination of the patient with an acute condition of the abdomen), it is rarely justified to defer analgesia until a diagnosis is made. In fact, a comfortable patient is better able to cooperate with diagnostic procedures.

28. A 26-year-old woman was robbed and beaten a month ago. She is returning to the clinic today for a followup assessment. The nurse will want to ask her which one of these questions? a. How are things going with the trial? b. How are things going with your job? c. Tell me about your recent engagement! d. Are you having any disturbing dreams?

d. Are you having any disturbing dreams? In posttraumatic stress disorder, the person has been exposed to a traumatic event. The traumatic event is persistently reexperienced by recurrent and intrusive, distressing recollections of the event, including images, thoughts, or perceptions; recurrent distressing dreams of the event; and acting or feeling as if the traumatic event were recurring.

24. During a class on religion and spirituality, the nurse is asked to define spirituality. Which answer is correct? Spirituality: a. Is a personal search to discover a supreme being. b. Is an organized system of beliefs concerning the cause, nature, and purpose of the universe. c. Is a belief that each person exists forever in some form, such as a belief in reincarnation or the afterlife. d. Arises out of each persons unique life experience and his or her personal effort to find purpose in life.

d. Arises out of each persons unique life experience and his or her personal effort to find purpose in life. Spirituality arises out of each persons unique life experience and his or her personal effort to find purpose and meaning in life. The other definitions reflect the concept of religion.

41. The nurse is performing a general survey of a patient. Which finding is considered normal? a. When standing, the patients base is narrow. b. The patient appears older than his stated age. c. Arm span (fingertip to fingertip) is greater than the height. d. Arm span (fingertip to fingertip) equals the patients height.

d. Arm span (fingertip to fingertip) equals the patients height. When performing the general survey, the patients arm span (fingertip to fingertip) should equal the patients height. An arm span that is greater than the persons height may indicate Marfan syndrome. The base should be wide when the patient is standing, and an older appearance than the stated age may indicate a history of a chronic illness or chronic alcoholism.

34. When examining an older adult, the nurse should use which technique? a. Avoid touching the patient too much. b. Attempt to perform the entire physical examination during one visit. c. Speak loudly and slowly because most aging adults have hearing deficits. d. Arrange the sequence of the examination to allow as few position changes as possible

d. Arrange the sequence of the examination to allow as few position changes as possible When examining the older adult, arranging the sequence of the examination to allow as few position changes as possible is best. Physical touch is especially important with the older person because other senses may be diminished.

42. During auscultation of a patients heart sounds, the nurse hears an unfamiliar sound. The nurse should: a. Document the findings in the patients record. b. Wait 10 minutes, and auscultate the sound again. c. Ask the patient how he or she is feeling. d. Ask another nurse to double check the finding.

d. Ask another nurse to double check the finding. If an abnormal finding is not familiar, then the nurse may ask another examiner to double check the finding. The other responses do not help identify the unfamiliar sound.

2. When measuring a patients weight, the nurse is aware of which of these guidelines? a. The patient is always weighed wearing only his or her undergarments. b. The type of scale does not matter, as long as the weights are similar from day to day. c. The patient may leave on his or her jacket and shoes as long as these are documented next to the weight. d. Attempts should be made to weigh the patient at approximately the same time of day, if a sequence of weights is necessary.

d. Attempts should be made to weigh the patient at approximately the same time of day, if a sequence of weights is necessary. standardized balance scale is used to measure weight. The patient should remove his or her shoes and heavy outer clothing. If a sequence of repeated weights is necessary, then the nurse should attempt to weigh the patient at approximately the same time of day and with the same types of clothing worn each time.

35. Which of the following reflects the traditional health and illness beliefs and practices of those of African heritage? Health is: a. Being rewarded for good behavior. b. The balance of the body and spirit. c. Maintained by wearing jade amulets. d. Being in harmony with nature.

d. Being in harmony with nature. The belief that health is being in harmony with nature reflects the health beliefs of those of African heritages. The other examples represent Iberian and Central and South American heritages, American-Indian heritages, and Asian heritages (See Table 2-3).

37. The nurse is preparing to assess a hospitalized patient who is experiencing significant shortness of breath. How should the nurse proceed with the assessment? a. The patient should lie down to obtain an accurate cardiac, respiratory, and abdominal assessment. b. A thorough history and physical assessment information should be obtained from the patients family member. c. A complete history and physical assessment should be immediately performed to obtain baseline information. d. Body areas appropriate to the problem should be examined and then the assessment completed after the problem has resolved.

d. Body areas appropriate to the problem should be examined and then the assessment completed after the problem has resolved. Both altering the position of the patient during the examination and collecting a mini database by examining the body areas appropriate to the problem may be necessary in this situation. An assessment may be completed later after the distress is resolved.

4. A pregnant woman is interested in breastfeeding her baby and asks several questions about the topic. Which information is appropriate for the nurse to share with her? a. Breastfeeding is best when also supplemented with bottle feedings. b. Babies who are breastfed often require supplemental vitamins. c. Breastfeeding is recommended for infants for the first 2 years of life. d. Breast milk provides the nutrients necessary for growth, as well as natural immunity.

d. Breast milk provides the nutrients necessary for growth, as well as natural immunity. Breastfeeding is recommended for full-term infants for the first year of life because breast milk is ideally formulated to promote normal infant growth and development, as well as natural immunity. The other statements are not correct.

15. Before auscultating the abdomen for the presence of bowel sounds on a patient, the nurse should: a. Warm the endpiece of the stethoscope by placing it in warm water. b. Leave the gown on the patient to ensure that he or she does not get chilled during the examination. c. Ensure that the bell side of the stethoscope is turned to the on position. d. Check the temperature of the room, and offer blankets to the patient if he or she feels cold.

d. Check the temperature of the room, and offer blankets to the patient if he or she feels cold. The examination room should be warm. If the patient shivers, then the involuntary muscle contractions can make it difficult to hear the underlying sounds. The end of the stethoscope should be warmed between the examiners hands, not with water. The nurse should never listen through a gown. The diaphragm of the stethoscope should be used to auscultate for bowel sounds.

30. The nurse is examining a 2-year-old child and asks, May I listen to your heart now? Which critique of the nurses technique is most accurate? a. Asking questions enhances the childs autonomy b. Asking the child for permission helps develop a sense of trust c. This question is an appropriate statement because children at this age like to have choices d. Children at this age like to say, No. The examiner should not offer a choice when no choice is available

d. Children at this age like to say, No. The examiner should not offer a choice when no choice is available Children at this age like to say, No. Choices should not be offered when no choice is really available. If the child says, No and the nurse does it anyway, then the nurse loses trust. Autonomy is enhanced by offering a limited option, Shall I listen to your heart next or your tummy?

18. A 5-year-old boy is being admitted to the hospital to have his tonsils removed. Which information should the nurse collect before this procedure? a. Childs birth weight b. Age at which he crawled c. Whether the child has had the measles d. Childs reactions to previous hospitalizations

d. Childs reactions to previous hospitalizations How the child reacted to previous hospitalizations and any complications should be assessed. If the child reacted poorly, then he or she may be afraid now and will need special preparation for the examination that is to follow. The other items are not significant for the procedure.

9. In using verbal responses to assist the patients narrative, some responses focus on the patients frame of reference and some focus on the health care providers perspective. An example of a verbal response that focuses on the health care providers perspective would be: a. Empathy. b. Reflection. c. Facilitation. d. Confrontation.

d. Confrontation. When the health care provider uses the response of confrontation, the frame of reference shifts from the patients perspective to the perspective of the health care provider, and the health care provider starts to express his or her own thoughts and feelings. Empathy, reflection, and facilitation responses focus on the patients frame of reference.

11. The nurse hears bilateral loud, long, and low tones when percussing over the lungs of a 4-year-old child. The nurse should: a. Palpate over the area for increased pain and tenderness. b. Ask the child to take shallow breaths, and percuss over the area again. c. Immediately refer the child because of an increased amount of air in the lungs. d. Consider this finding as normal for a child this age, and proceed with the examination.

d. Consider this finding as normal for a child this age, and proceed with the examination. Percussion notes that are loud in amplitude, low in pitch, of a booming quality, and long in duration are normal over a childs lung.

2. During a class on the aspects of culture, the nurse shares that culture has four basic characteristics. Which statement correctly reflects one of these characteristics? a. Cultures are static and unchanging, despite changes around them. b. Cultures are never specific, which makes them hard to identify. c. Culture is most clearly reflected in a persons language and behavior. d. Culture adapts to specific environmental factors and available natural resources.

d. Culture adapts to specific environmental factors and available natural resources. Culture has four basic characteristics. Culture adapts to specific conditions related to environmental and technical factors and to the availability of natural resources, and it is dynamic and ever changing. Culture is learned from birth through the process of language acquisition and socialization, but it is not most clearly reflected in ones language and behavior.

22. The nurse is obtaining a health history on an 87-year-old woman. Which of the following areas of questioning would be most useful at this time? a. Obstetric history b. Childhood illnesses c. General health for the past 20 years d. Current health promotion activities

d. Current health promotion activities It is important for the nurse to recognize positive health measures, such as what the person has been doing to help him or herself stay well and to live to an older age. The other responses are not pertinent to a patient of this age.

15. A patient has been admitted to the hospital with vertebral fractures related to osteoporosis. She is in extreme pain. This type of pain would be classified as: a. Referred. b. Cutaneous. c. Visceral. d. Deep somatic.

d. Deep somatic. Deep somatic pain comes from sources such as the blood vessels, joints, tendons, muscles, and bone. Referred pain is felt at one site but originates from another location. Cutaneous pain is derived from the skin surface and subcutaneous tissues. Visceral pain originates from the larger, interior organs.

8. A patient tells the nurse that he is allergic to penicillin. What would be the nurses best response to this information? a. Are you allergic to any other drugs? b. How often have you received penicillin? c. Ill write your allergy on your chart so you wont receive any penicillin. d. Describe what happens to you when you take penicillin.

d. Describe what happens to you when you take penicillin. Note both the allergen (medication, food, or contact agent, such as fabric or environmental agent) and the reaction (rash, itching, runny nose, watery eyes, or difficulty breathing). With a drug, this symptom should not be a side effect but a true allergic reaction

16. The mother of a 16-month-old toddler tells the nurse that her daughter has an earache. What would be an appropriate response? a. Maybe she is just teething. b. I will check her ear for an ear infection. c. Are you sure she is really having pain? d. Describe what she is doing to indicate she is having pain.

d. Describe what she is doing to indicate she is having pain. With a very young child, the parent is asked, How do you know the child is in pain? A young child pulling at his or her ears should alert parents to the childs ear pain. Statements about teething and questioning whether the child is really having pain do not explore the symptoms, which should be done before a physical examination.

17. The nurse is preparing to use an otoscope for an examination. Which statement is true regarding the otoscope? The otoscope: a. Is often used to direct light onto the sinuses. b. Uses a short, broad speculum to help visualize the ear. c. Is used to examine the structures of the internal ear. d. Directs light into the ear canal and onto the tympanic membrane

d. Directs light into the ear canal and onto the tympanic membrane The otoscope directs light into the ear canal and onto the tympanic membrane that divides the external and middle ear. A short, broad speculum is used to visualize the nares.

26. The nurse is preparing to measure fat and lean body mass and bone mineral density. Which tool is appropriate? a. Measuring tape b. Skinfold calipers c. Bioelectrical impedance analysis (BIA) d. Dual-energy x-ray absorptiometry (DEXA)

d. Dual-energy x-ray absorptiometry (DEXA) DEXA measures both bone mineral density and fat and lean body mass. BIA measures fat and lean body mass but not bone mineral density. A measuring tape measures distance or length, and skinfold calipers are used to determine skinfold thickness.

3. A 4-year-old boy is brought to the emergency department by his mother. She says he points to his stomach and says, It hurts so bad. Which pain assessment tool would be the best choice when assessing this childs pain? a. Descriptor Scale b. Numeric rating scale c. Brief Pain Inventory d. Faces Pain ScaleRevised (FPS-R)

d. Faces Pain ScaleRevised (FPS-R) Rating scales can be introduced at the age of 4 or 5 years. The FPS-R is designed for use by children and asks the child to choose a face that shows how much hurt (or pain) you have now. Young children should not be asked to rate pain by using numbers.

13. The nurse is discussing appropriate foods with the mother of a 3-year-old child. Which of these foods are recommended? a. Foods that the child will eat, no matter what they are b. Foods easy to hold such as hot dogs, nuts, and grapes c. Any foods, as long as the rest of the family is also eating them d. Finger foods and nutritious snacks that cannot cause choking

d. Finger foods and nutritious snacks that cannot cause choking Small portions, finger foods, simple meals, and nutritious snacks help improve the dietary intake of young children. Foods likely to be aspirated should be avoided (e.g., hot dogs, nuts, grapes, round candies, popcorn).

20. When observing a patients verbal and nonverbal communication, the nurse notices a discrepancy. Which statement is true regarding this situation? The nurse should: a. Ask someone who knows the patient well to help interpret this discrepancy. b. Focus on the patients verbal message, and try to ignore the nonverbal behaviors. c. Try to integrate the verbal and nonverbal messages and then interpret them as an average. d. Focus on the patients nonverbal behaviors, because these are often more reflective of a patients true feelings.

d. Focus on the patients nonverbal behaviors, because these are often more reflective of a patients true feelings. When nonverbal and verbal messages are congruent, the verbal message is reinforced. When they are incongruent, the nonverbal message tends to be the true one because it is under less conscious control. Thus studying the nonverbal messages of the patients and examiners and understanding their meanings are important. The other statements are not true.

8. A patient is asked to indicate on a form how many times he eats a specific food. This method describes which of these tools for obtaining dietary information? a. Food diary b. Calorie count c. 24-hour recall d. Food-frequency questionnaire

d. Food-frequency questionnaire With this tool, information is collected on how many times per day, week, or month the individual eats particular foods, which provides an estimate of usual intake.

7. A patient has had arthritic pain in her hips for several years since a hip fracture. She is able to move around in her room and has not offered any complaints so far this morning. However, when asked, she states that her pain is bad this morning and rates it at an 8 on a 1-to-10 scale. What does the nurse suspect? The patient: a. Is addicted to her pain medications and cannot obtain pain relief. b. Does not want to trouble the nursing staff with her complaints. c. Is not in pain but rates it high to receive pain medication. d. Has experienced chronic pain for years and has adapted to it.

d. Has experienced chronic pain for years and has adapted to it. Persons with chronic pain typically try to give little indication that they are in pain and, over time, adapt to the pain. As a result, they are at risk for underdetection.

34. When planning a cultural assessment, the nurse should include which component? a. Family history b. Chief complaint c. Medical history d. Health-related beliefs

d. Health-related beliefs Health-related beliefs and practices are one component of a cultural assessment. The other items reflect other aspects of the patients history.

16. The nurse is performing a mental status assessment on a 5-year-old girl. Her parents are undergoing a bitter divorce and are worried about the effect it is having on their daughter. Which action or statement might lead the nurse to be concerned about the girls mental status? a. She clings to her mother whenever the nurse is in the room. b. She appears angry and will not make eye contact with the nurse. c. Her mother states that she has begun to ride a tricycle around their yard. d. Her mother states that her daughter prefers to play with toddlers instead of kids her own age while in daycare.

d. Her mother states that her daughter prefers to play with toddlers instead of kids her own age while in daycare. The mental status assessment of infants and children covers behavioral, cognitive, and psychosocial development and examines how the child is coping with his or her environment. Essentially, the nurse should follow the same Association for Behavioral and Cognitive Therapies (ABCT) guidelines as those for the adult, with special consideration for developmental milestones. The best examination technique arises from a thorough knowledge of the developmental milestones (described in Chapter 2). Abnormalities are often problems of omission (e.g., the child does not achieve a milestone as expected).

31. A man arrives at the clinic for his annual wellness physical. He is experiencing no acute health problems. Which question or statement by the nurse is most appropriate when beginning the interview? a. How is your family? b. How is your job? c. Tell me about your hypertension. d. How has your health been since your last visit?

d. How has your health been since your last visit? Open-ended questions are used for gathering narrative information. This type of questioning should be used to begin the interview, to introduce a new section of questions, and whenever the person introduces a new topic.

5. A 29-year-old woman tells the nurse that she has excruciating pain in her back. Which would be the nurses appropriate response to the womans statement? a. How does your family react to your pain? b. The pain must be terrible. You probably pinched a nerve. c. Ive had back pain myself, and it can be excruciating. d. How would you say the pain affects your ability to do your daily activities?

d. How would you say the pain affects your ability to do your daily activities? The symptom of pain is difficult to quantify because of individual interpretation. With pain, adjectives should be avoided and the patient should be asked how the pain affects his or her daily activities. The other responses are not appropriate.

7. During an assessment, the nurse asks a female patient, How many alcoholic drinks do you have a week? Which answer by the patient would indicate at-risk drinking? a. I may have one or two drinks a week. b. I usually have three or four drinks a week. c. Ill have a glass or two of wine every now and then. d. I have seven or eight drinks a week, but I never get drunk.

d. I have seven or eight drinks a week, but I never get drunk. For women, having seven or more drinks a week or three or more drinks per occasion is considered at-risk drinking, according to the National Institute on Alcohol Abuse and Alcoholism.

17. The nurse is assessing orientation in a 79-year-old patient. Which of these responses would lead the nurse to conclude that this patient is oriented? a. I know my name is John. I couldnt tell you where I am. I think it is 2010, though. b. I know my name is John, but to tell you the truth, I get kind of confused about the date. c. I know my name is John; I guess Im at the hospital in Spokane. No, I dont know the date. d. I know my name is John. I am at the hospital in Spokane. I couldnt tell you what date it is, but I know that it is February of a new year2010.

d. I know my name is John. I am at the hospital in Spokane. I couldnt tell you what date it is, but I know that it is February of a new year2010. Many aging persons experience social isolation, loss of structure without a job, a change in residence, or some short-term memory loss. These factors affect orientation, and the person may not provide the precise date or complete name of the agency. You may consider aging persons oriented if they generally know where they are and the present period. They should be considered oriented to time if the year and month are correctly stated. Orientation to place is accepted with the correct identification of the type of setting (e.g., hospital) and the name of the town.

4. When discussing the use of the term subculture, the nurse recognizes that it is best described as: a. Fitting as many people into the majority culture as possible. b. Defining small groups of people who do not want to be identified with the larger culture. c. Singling out groups of people who suffer differential and unequal treatment as a result of cultural variations. d. Identifying fairly large groups of people with shared characteristics that are not common to all members of a culture.

d. Identifying fairly large groups of people with shared characteristics that are not common to all members of a culture. Within cultures, groups of people share different beliefs, values, and attitudes. Differences occur because of ethnicity, religion, education, occupation, age, and gender. When such groups function within a large culture, they are referred to as subcultural groups.

13. Which of these responses might the nurse expect during a functional assessment of a patient whose leg is in a cast? a. I broke my right leg in a car accident 2 weeks ago. b. The pain is decreasing, but I still need to take acetaminophen. c. I check the color of my toes every evening just like I was taught. d. Im able to transfer myself from the wheelchair to the bed without help.

d. Im able to transfer myself from the wheelchair to the bed without help. Functional assessment measures a persons self-care ability in the areas of general physical health or absence of illness. The other statements concern health or illness issues.

24. A patient has been diagnosed with schizophrenia. During a recent interview, he shows the nurse a picture of a man holding a decapitated head. He describes this picture as horrifying but then laughs loudly at the content. This behavior is a display of: a. Confusion b. Ambivalence c. Depersonalization d. Inappropriate affect

d. Inappropriate affect An inappropriate affect is an affect clearly discordant with the content of the persons speech. (See Table 5-5 for the definitions of the other terms.)

27. After a symptom is recognized, the first effort at treatment is often self-care. Which of the following statements about self-care is true? Self-care is: a. Not recognized as valuable by most health care providers. b. Usually ineffective and may delay more effective treatment. c. Always less expensive than biomedical alternatives. d. Influenced by the accessibility of over-the-counter medicines.

d. Influenced by the accessibility of over-the-counter medicines. After a symptom is identified, the first effort at treatment is often self-care. The availability of over-the-counter medications, the relatively high literacy level of Americans, and the influence of the mass media in communicating health-related information to the general population have contributed to the high percentage of cases of self-treatment.

26. The nurse is preparing to do a functional assessment. Which statement best describes the purpose of a functional assessment? a. The functional assessment assesses how the individual is coping with life at home. b. It determines how children are meeting developmental milestones. c. The functional assessment can identify any problems with memory the individual may be experiencing. d. It helps determine how a person is managing day-to-day activities.

d. It helps determine how a person is managing day-to-day activities. The functional assessment measures how a person manages day-to-day activities. The other answers do not reflect the purpose of a functional assessment.

21. A 63-year-old Chinese-American man enters the hospital with complaints of chest pain, shortness of breath, and palpitations. Which statement most accurately reflects the nurses best course of action? a. The nurse should focus on performing a full cardiac assessment. b. The nurse should focus on psychosomatic complaints because the patient has just learned that his wife has cancer. c. This patient is not in any danger at present; therefore, the nurse should send him home with instructions to contact his physician. d. It is unclear what is happening with this patient; consequently, the nurse should perform an assessment in both the physical and the psychosocial realms.

d. It is unclear what is happening with this patient; consequently, the nurse should perform an assessment in both the physical and the psychosocial realms. Wide cultural variations exist in the manner in which certain symptoms and disease conditions are perceived, diagnosed, labeled, and treated. Chinese-Americans sometimes convert mental experiences or states into bodily symptoms (e.g., complaining of cardiac symptoms because the center of emotion in the Chinese culture is the heart).

38. A female nurse is interviewing a male patient who is near the same age as the nurse. During the interview, the patient makes an overtly sexual comment. The nurses best reaction would be: a. Stop that immediately! b. Oh, you are too funny. Lets keep going with the interview. c. Do you really think I would be interested? d. It makes me uncomfortable when you talk that way. Please stop.

d. It makes me uncomfortable when you talk that way. Please stop. The nurses response must make it clear that she is a health professional who can best care for the person by maintaining a professional relationship. At the same time, the nurse should communicate that he or she accepts the person and understands the persons need to be self-assertive but that sexual advances cannot be tolerated.

3. A 59-year-old patient tells the nurse that he has ulcerative colitis. He has been having black stools for the last 24 hours. How would the nurse best document his reason for seeking care? a. J.M. is a 59-year-old man seeking treatment for ulcerative colitis. b. J.M. came into the clinic complaining of having black stools for the past 24 hours. c. J.M. is a 59-year-old man who states that he has ulcerative colitis and wants it checked. d. J.M. is a 59-year-old man who states that he has been having black stools for the past 24 hours.

d. J.M. is a 59-year-old man who states that he has been having black stools for the past 24 hours. The reason for seeking care is a brief spontaneous statement in the persons own words that describes the reason for the visit. It states one (possibly two) signs or symptoms and their duration. It is enclosed in quotation marks to indicate the persons exact words.

26. A 20-year-old construction worker has been brought into the emergency department with heat stroke. He has delirium as a result of a fluid and electrolyte imbalance. For the mental status examination, the nurse should first assess the patients: a. Affect and mood b. Memory and affect c. Language abilities d. Level of consciousness and cognitive abilities

d. Level of consciousness and cognitive abilities Delirium is a disturbance of consciousness (i.e., reduced clarity of awareness of the environment) with reduced ability to focus, sustain, or shift attention. Delirium is not an alteration in mood, affect, or language abilities.

20. In obtaining a review of systems on a healthy 7-year-old girl, the health care provider knows that it would be important to include the: a. Last glaucoma examination. b. Frequency of breast self-examinations. c. Date of her last electrocardiogram. d. Limitations related to her involvement in sports activities.

d. Limitations related to her involvement in sports activities. When reviewing the cardiovascular system, the health care provider should ask whether any activity is limited or whether the child can keep up with her peers. The other items are not appropriate for a child this age.

7. For the first time, the nurse is seeing a patient who has no history of nutrition-related problems. The initial nutritional screening should include which activity? a. Calorie count of nutrients b. Anthropometric measures c. Complete physical examination d. Measurement of weight and weight history

d. Measurement of weight and weight history Parameters used for nutrition screening typically include weight and weight history, conditions associated with increased nutritional risk, diet information, and routine laboratory data. The other responses reflect a more indepth assessment rather than a screening.

9. A 19-year-old woman comes to the clinic at the insistence of her brother. She is wearing black combat boots and a black lace nightgown over the top of her other clothes. Her hair is dyed pink with black streaks throughout. She has several pierced holes in her nares and ears and is wearing an earring through her eyebrow and heavy black makeup. The nurse concludes that: a. She probably does not have any problems. b. She is only trying to shock people and that her dress should be ignored. c. She has a manic syndrome because of her abnormal dress and grooming. d. More information should be gathered to decide whether her dress is appropriate.

d. More information should be gathered to decide whether her dress is appropriate. Grooming and hygiene should be notedthe person is clean and well groomed, hair is neat and clean, women have moderate or no makeup, and men are shaved or their beards or moustaches are well groomed. Care should be taken when interpreting clothing that is disheveled, bizarre, or in poor repair because these sometimes reflect the persons economic status or a deliberate fashion trend.

13. A man has been admitted to the observation unit for observation after being treated for a large cut on his forehead. As the nurse works through the interview, one of the standard questions has to do with alcohol, tobacco, and drug use. When the nurse asks him about tobacco use, he states, I quit smoking after my wife died 7 years ago. However, the nurse notices an open pack of cigarettes in his shirt pocket. Using confrontation, the nurse could say: a. Mr. K., I know that you are lying. b. Mr. K., come on, tell me how much you smoke. c. Mr. K., I didnt realize your wife had died. It must be difficult for you at this time. Please tell me more about that. d. Mr. K., you have said that you dont smoke, but I see that you have an open pack of cigarettes in your pocket.

d. Mr. K., you have said that you dont smoke, but I see that you have an open pack of cigarettes in your pocket. In the case of confrontation, a certain action, feeling, or statement has been observed, and the nurse now focuses the patients attention on it. The nurse should give honest feedback about what is seen or felt. Confrontation may focus on a discrepancy, or the nurse may confront the patient when parts of the story are inconsistent. The other statements are not appropriate.

8. A patient has finished giving the nurse information about the reason he is seeking care. When reviewing the data, the nurse finds that some information about past hospitalizations is missing. At this point, which statement by the nurse would be most appropriate to gather these data? a. Mr. Y., at your age, surely you have been hospitalized before! b. Mr. Y., I just need permission to get your medical records from County Medical. c. Mr. Y., you mentioned that you have been hospitalized on several occasions. Would you tell me more about that? d. Mr. Y., I just need to get some additional information about your past hospitalizations. When was the last time you were admitted for chest pain?

d. Mr. Y., I just need to get some additional information about your past hospitalizations. When was the last time you were admitted for chest pain? The nurse should use direct questions after the persons opening narrative to fill in any details he or she left out. The nurse also should use direct questions when specific facts are needed, such as when asking about past health problems or during the review of systems.

6. A woman has just entered the emergency department after being battered by her husband. The nurse needs to get some information from her to begin treatment. What is the best choice for an opening phase of the interview with this patient? a. Hello, Nancy, my name is Mrs. C. b. Hello, Mrs. H., my name is Mrs. C. It sure is cold today! c. Mrs. H., my name is Mrs. C. How are you? d. Mrs. H., my name is Mrs. C. Ill need to ask you a few questions about what happened.

d. Mrs. H., my name is Mrs. C. Ill need to ask you a few questions about what happened. Address the person by using his or her surname. The nurse should introduce him or herself and give the reason for the interview. Friendly small talk is not needed to build rapport.

8. The nurse is reviewing the principles of pain. Which type of pain is due to an abnormal processing of the pain impulse through the peripheral or central nervous system? a. Visceral b. Referred c. Cutaneous d. Neuropathic

d. Neuropathic Neuropathic pain implies an abnormal processing of the pain message. The other types of pain are named according to their sources.

5. A woman who has just discovered that she is pregnant is in the clinic for her first obstetric visit. She asks the nurse, How many drinks a day is safe for my baby? The nurses best response is: a. You should limit your drinking to once or twice a week. b. Its okay to have up to two glasses of wine a day. c. As long as you avoid getting drunk, you should be safe. d. No amount of alcohol has been determined to be safe during pregnancy

d. No amount of alcohol has been determined to be safe during pregnancy No amount of alcohol has been determined to be safe for pregnant women. The potential adverse effects of alcohol use on the fetus are well known; women who are pregnant should be screened for alcohol use, and abstinence should be recommended.

15. Which of these individuals would the nurse consider at highest risk for a suicide attempt? a. Man who jokes about death b. Woman who, during a past episode of major depression, attempted suicide c. Adolescent who just broke up with her boyfriend and states that she would like to kill herself d. Older adult man who tells the nurse that he is going to join his wife in heaven tomorrow and plans to use a gun

d. Older adult man who tells the nurse that he is going to join his wife in heaven tomorrow and plans to use a gun When the person expresses feelings of sadness, hopelessness, despair, or grief, assessing any possible risk of physical harm to him or herself is important. The interview should begin with more general questions. If the nurse hears affirmative answers, then he or she should continue with more specific questions. A precise suicide plan to take place in the next 24 to 48 hours with use of a lethal method constitutes high risk.

29. In the hot/cold theory, illnesses are believed to be caused by hot or cold entering the body. Which of these patient conditions is most consistent with a cold condition? a. Patient with diabetes and renal failure b. Teenager with an abscessed tooth c. Child with symptoms of itching and a rash d. Older man with gastrointestinal discomfort

d. Older man with gastrointestinal discomfort Illnesses believed to be caused by cold entering the body include earache, chest cramps, gastrointestinal discomfort, rheumatism, and tuberculosis. Those illnesses believed to be caused by heat, or overheating, include sore throats, abscessed teeth, rashes, and kidney disorders.

7. During an interview, the nurse states, You mentioned having shortness of breath. Tell me more about that. Which verbal skill is used with this statement? a. Reflection b. Facilitation c. Direct question d. Open-ended question

d. Open-ended question The open-ended question asks for narrative information. It states the topic to be discussed but only in general terms. The nurse should use it to begin the interview, to introduce a new section of questions, and whenever the person introduces a new topic.

5. The nurse asks, I would like to ask you some questions about your health and your usual daily activities so that we can better plan your stay here. This question is found at the __________ phase of the interview process. a. Summary b. Closing c. Body d. Opening or introduction

d. Opening or introduction When gathering a complete history, the nurse should give the reason for the interview during the opening or introduction phase of the interview, not during or at the end of the interview.

20. The nurse is preparing to perform a physical assessment. The correct action by the nurse is reflected by which statement? The nurse: a. Performs the examination from the left side of the bed. b. Examines tender or painful areas first to help relieve the patients anxiety. c. Follows the same examination sequence, regardless of the patients age or condition. d. Organizes the assessment to ensure that the patient does not change positions too often.

d. Organizes the assessment to ensure that the patient does not change positions too often. The steps of the assessment should be organized to ensure that the patient does not change positions too often. The sequence of the steps of the assessment may differ, depending on the age of the person and the examiners preference. Tender or painful areas should be assessed last.

6. In recording the childhood illnesses of a patient who denies having had any, which note by the nurse would be most accurate? a. Patient denies usual childhood illnesses. b. Patient states he was a very healthy child. c. Patient states his sister had measles, but he didnt. d. Patient denies measles, mumps, rubella, chickenpox, pertussis, and strep throat.

d. Patient denies measles, mumps, rubella, chickenpox, pertussis, and strep throat. Childhood illnesses include measles, mumps, rubella, chickenpox, pertussis, and strep throat. Avoid recording usual childhood illnesses because an illness common in the persons childhood may be unusual today (e.g., measles).

26. While measuring a patients blood pressure, the nurse recalls that certain factors, such as __________, help determine blood pressure. a. Pulse rate b. Pulse pressure c. Vascular output d. Peripheral vascular resistance

d. Peripheral vascular resistance The level of blood pressure is determined by five factors: cardiac output, peripheral vascular resistance, volume of circulating blood, viscosity, and elasticity of the vessel walls.

30. A 23-year-old patient in the clinic appears anxious. Her speech is rapid, and she is fidgety and in constant motion. Which of these questions or statements would be most appropriate for the nurse to use in this situation to assess attention span? a. How do you usually feel? Is this normal behavior for you? b. I am going to say four words. In a few minutes, I will ask you to recall them. c. Describe the meaning of the phrase, Looking through rose-colored glasses. d. Pick up the pencil in your left hand, move it to your right hand, and place it on the table.

d. Pick up the pencil in your left hand, move it to your right hand, and place it on the table. Attention span is evaluated by assessing the individuals ability to concentrate and complete a thought or task without wandering. Giving a series of directions to follow is one method used to assess attention span.

33. When examining a 16-year-old male teenager, the nurse should: a. Discuss health teaching with the parent because the teen is unlikely to be interested in promoting wellness. b. Ask his parent to stay in the room during the history and physical examination to answer any questions and to alleviate his anxiety. c. Talk to him the same manner as one would talk to a younger child because a teens level of understanding may not match his or her speech. d. Provide feedback that his body is developing normally, and discuss the wide variation among teenagers on the rate of growth and development.

d. Provide feedback that his body is developing normally, and discuss the wide variation among teenagers on the rate of growth and development. During the examination, the adolescent needs feedback that his or her body is healthy and developing normally. The adolescent has a keen awareness of body image and often compares him or herself with peers. Apprise the adolescent of the wide variation among teenagers on the rate of growth and development.

2. When assessing a patients nutritional status, the nurse recalls that the best definition of optimal nutritional status is sufficient nutrients that: a. Are in excess of daily body requirements. b. Provide for the minimum body needs. c. Provide for daily body requirements but do not support increased metabolic demands. d. Provide for daily body requirements and support increased metabolic demands.

d. Provide for daily body requirements and support increased metabolic demands. Optimal nutritional status is achieved when sufficient nutrients are consumed to support day-to-day body needs and any increased metabolic demands resulting from growth, pregnancy, or illness.

15. When assessing a 75-year-old patient who has asthma, the nurse notes that he assumes a tripod position, leaning forward with arms braced on the chair. On the basis of this observation, the nurse should: a. Assume that the patient is eager and interested in participating in the interview. b. Evaluate the patient for abdominal pain, which may be exacerbated in the sitting position. c. Assume that the patient is having difficulty breathing and assist him to a supine position. d. Recognize that a tripod position is often used when a patient is having respiratory difficulties.

d. Recognize that a tripod position is often used when a patient is having respiratory difficulties. Assuming a tripod positionleaning forward with arms braced on chair armsoccurs with chronic pulmonary disease. The other actions or assumptions are not correct.

18. An examiner is using an ophthalmoscope to examine a patients eyes. The patient has astigmatism and is nearsighted. The use of which of these techniques would indicate that the examination is being correctly performed? a. Using the large full circle of light when assessing pupils that are not dilated b. Rotating the lens selector dial to the black numbers to compensate for astigmatism c. Using the grid on the lens aperture dial to visualize the external structures of the eye d. Rotating the lens selector dial to bring the object into focus

d. Rotating the lens selector dial to bring the object into focus The ophthalmoscope is used to examine the internal eye structures. It can compensate for nearsightedness or farsightedness, but it will not correct for astigmatism. The grid is used to assess size and location of lesions on the fundus. The large full spot of light is used to assess dilated pupils. Rotating the lens selector dial brings the object into focus.

4. When assessing aging adults, the nurse knows that one of the first things that should be assessed before making judgments about their mental status is: a. Presence of phobias b. General intelligence c. Presence of irrational thinking patterns d. Sensory-perceptive abilities

d. Sensory-perceptive abilities Age-related changes in sensory perception can affect mental status. For example, vision loss (as detailed in Chapter 14) may result in apathy, social isolation, and depression. Hearing changes are common in older adults, which produces frustration, suspicion, and social isolation and makes the person appear confused.

29. A female American Indian has come to the clinic for follow-up diabetic teaching. During the interview, the nurse notices that she never makes eye contact and speaks mostly to the floor. Which statement is true regarding this situation? a. The woman is nervous and embarrassed. b. She has something to hide and is ashamed. c. The woman is showing inconsistent verbal and nonverbal behaviors. d. She is showing that she is carefully listening to what the nurse is saying.

d. She is showing that she is carefully listening to what the nurse is saying. Eye contact is perhaps among the most culturally variable nonverbal behaviors. Asian, American Indian, Indochinese, Arabian, and Appalachian people may consider direct eye contact impolite or aggressive, and they may avert their eyes during the interview. American Indians often stare at the floor during the interview, which is a culturally appropriate behavior, indicating that the listener is paying close attention to the speaker.

5. A 60-year-old woman has developed reflexive sympathetic dystrophy after arthroscopic repair of her shoulder. A key feature of this condition is that the: a. Affected extremity will eventually regain its function. b. Pain is felt at one site but originates from another location. c. Patients pain will be associated with nausea, pallor, and diaphoresis. d. Slightest touch, such as a sleeve brushing against her arm, causes severe and intense pain.

d. Slightest touch, such as a sleeve brushing against her arm, causes severe and intense pain. A key feature of reflexive sympathetic dystrophy is that a typically innocuous stimulus can create a severe, intensely painful response. The affected extremity becomes less functional over time.

32. The nurse recognizes that categories such as ethnicity, gender, and religion illustrate the concept of: a. Family. b. Cultures. c. Spirituality. d. Subcultures.

d. Subcultures. Within cultures, groups of people share different beliefs, values, and attitudes. Differences occur because of ethnicity, religion, education, occupation, age, and gender. When such groups function within a large culture, they are referred to as subcultural groups.

36. During the interview portion of data collection, the nurse collects __________ data. a. Physical b. Historical c. Objective d. Subjective

d. Subjective The interview is the first, and really the most important, part of data collection. During the interview, the nurse collects subjective data; that is, what the person says about him or herself.

6. The nurse is assessing a patients pain. The nurse knows that the most reliable indicator of pain would be the: a. Patients vital signs. b. Physical examination. c. Results of a computerized axial tomographic scan. d. Subjective report.

d. Subjective report. The subjective report is the most reliable indicator of pain. Physical examination findings can lend support, but the clinician cannot exclusively base the diagnosis of pain on physical assessment findings.

1. As a mandatory reporter of elder abuse, which must be present before a nurse should notify the authorities? a. Statements from the victim b. Statements from witnesses c. Proof of abuse and/or neglect d. Suspicion of elder abuse and/or neglect

d. Suspicion of elder abuse and/or neglect Many health care workers are under the erroneous assumption that proof is required before notification of suspected abuse can occur. Only the suspicion of elder abuse or neglect is necessary.

23. During an examination of a 3-year-old child, the nurse will need to take her blood pressure. What might the nurse do to try to gain the childs full cooperation? a. Tell the child that the blood pressure cuff is going to give her arm a big hug. b. Tell the child that the blood pressure cuff is asleep and cannot wake up. c. Give the blood pressure cuff a name and refer to it by this name during the assessment. d. Tell the child that by using the blood pressure cuff, we can see how strong her muscles are.

d. Tell the child that by using the blood pressure cuff, we can see how strong her muscles are. Take the time to give a short, simple explanation with a concrete explanation for any unfamiliar equipment that will be used on the child. Preschoolers are animistic; they imagine inanimate objects can come alive and have human characteristics. Thus a blood pressure cuff can wake up and bite or pinch.

5. The nurse is preparing to assess a patients abdomen by palpation. How should the nurse proceed? a. Palpation of reportedly tender areas are avoided because palpation in these areas may cause pain. b. Palpating a tender area is quickly performed to avoid any discomfort that the patient may experience. c. The assessment begins with deep palpation, while encouraging the patient to relax and to take deep breaths. d. The assessment begins with light palpation to detect surface characteristics and to accustom the patient to being touched.

d. The assessment begins with light palpation to detect surface characteristics and to accustom the patient to being touched. Light palpation is initially performed to detect any surface characteristics and to accustom the person to being touched. Tender areas should be palpated last, not first.

42. The nurse is assessing children in a pediatric clinic. Which statement is true regarding the measurement of blood pressure in children? a. Blood pressure guidelines for children are based on age. b. Phase II Korotkoff sounds are the best indicator of systolic blood pressure in children. c. Using a Doppler device is recommended for accurate blood pressure measurements until adolescence. d. The disappearance of phase V Korotkoff sounds can be used for the diastolic reading in children.

d. The disappearance of phase V Korotkoff sounds can be used for the diastolic reading in children. The disappearance of phase V Korotkoff sounds can be used for the diastolic reading in children, as well as in adults.

9. The nurse is using the danger assessment (DA) tool to evaluate the risk of homicide. Which of these statements best describes its use? a. The DA tool is to be administered by law enforcement personnel. b. The DA tool should be used in every assessment of suspected abuse. c. The number of yes answers indicates the womans understanding of her situation. d. The higher the number of yes answers, the more serious the danger of the womans situation.

d. The higher the number of yes answers, the more serious the danger of the womans situation. No predetermined cutoff scores exist on the DA. The higher the number yes answers, the more serious the danger of the womans situation. The use of this tool is not limited to law enforcement personnel and is not required in every case of suspected abuse.

29. During an assessment, the nurse uses the CAGE test. The patient answers yes to two of the questions. What could this be indicating? a. The patient is an alcoholic. b. The patient is annoyed at the questions. c. The patient should be thoroughly examined for possible alcohol withdrawal symptoms. d. The nurse should suspect alcohol abuse and continue with a more thorough substance abuse assessment.

d. The nurse should suspect alcohol abuse and continue with a more thorough substance abuse assessment. The CAGE test is known as the cut down, annoyed, guilty, and eye-opener test. If a person answers yes to two or more of the four CAGE questions, then the nurse should suspect alcohol abuse and continue with a more complete substance abuse assessment.

28. The nurse is reviewing the hot/cold theory of health and illness. Which statement best describes the basic tenets of this theory? a. The causation of illness is based on supernatural forces that influence the humors of the body. b. Herbs and medicines are classified on their physical characteristics of hot and cold and the humors of the body. c. The four humors of the body consist of blood, yellow bile, spiritual connectedness, and social aspects of the individual. d. The treatment of disease consists of adding or subtracting cold, heat, dryness, or wetness to restore the balance of the humors of the body.

d. The treatment of disease consists of adding or subtracting cold, heat, dryness, or wetness to restore the balance of the humors of the body. The hot/cold theory of health and illness is based on the four humors of the body: blood, phlegm, black bile, and yellow bile. These humors regulate the basic bodily functions, described in terms of temperature, dryness, and moisture. The treatment of disease consists of adding or subtracting cold, heat, dryness, or wetness to restore the balance of the humors. The other statements are not correct.

49. When checking for proper blood pressure cuff size, which guideline is correct? a. The standard cuff size is appropriate for all sizes. b. The length of the rubber bladder should equal 80% of the arm circumference. c. The width of the rubber bladder should equal 80% of the arm circumference. d. The width of the rubber bladder should equal 40% of the arm circumference.

d. The width of the rubber bladder should equal 40% of the arm circumference. The width of the rubber bladder should equal 40% of the circumference of the persons arm. The length of the bladder should equal 80% of this circumference.

28. A patient is describing his symptoms to the nurse. Which of these statements reflects a description of the setting of his symptoms? a. It is a sharp, burning pain in my stomach. b. I also have the sweats and nausea when I feel this pain. c. I think this pain is telling me that something bad is wrong with me. d. This pain happens every time I sit down to use the computer.

d. This pain happens every time I sit down to use the computer. The setting describes where the person is or what the person is doing when the symptom starts. Describing the pain as sharp and burning reflects the character or quality of the pain; stating that the pain is telling the patient that something bad is wrong with him reflects the patients perception of the pain; and describing the sweats and nausea reflects associated factors that occur with the pain.

The nurse is preparing to conduct a health history. Which of these statements best describes the purpose of a health history? a. To provide an opportunity for interaction between the patient and the nurse b. To provide a form for obtaining the patients biographic information c. To document the normal and abnormal findings of a physical assessment d. To provide a database of subjective information about the patients past and current health

d. To provide a database of subjective information about the patients past and current health The purpose of the health history is to collect subjective datawhat the person says about him or herself. The other options are not correct.

21. During an interview, a parent of a hospitalized child is sitting in an open position. As the interviewer begins to discuss his sons treatment, however, he suddenly crosses his arms against his chest and crosses his legs. This changed posture would suggest that the parent is: a. Simply changing positions. b. More comfortable in this position. c. Tired and needs a break from the interview. d. Uncomfortable talking about his sons treatment.

d. Uncomfortable talking about his sons treatment. The persons position is noted. An open position with the extension of large muscle groups shows relaxation, physical comfort, and a willingness to share information. A closed position with the arms and legs crossed tends to look defensive and anxious. Any change in posture should be noted. If a person in a relaxed position suddenly tenses, then this change in posture suggests possible discomfort with the new topic.

24. A 90-year-old patient tells the nurse that he cannot remember the names of the medications he is taking or for what reason he is taking them. An appropriate response from the nurse would be: a. Can you tell me what they look like? b. Dont worry about it. You are only taking two medications. c. How long have you been taking each of the pills? d. Would you have a family member bring in your medications?

d. Would you have a family member bring in your medications? The person may not know the drug name or purpose. When this occurs, ask the person or a family member to bring in the drug to be identified. The other responses would not help to identify the medications.

7. The nurse manager is explaining culturally competent care during a staff meeting. Which statement accurately describes the concept of culturally competent care? The caregiver: a. Is able to speak the patients native language. b. Possesses some basic knowledge of the patients cultural background. c. Applies the proper background knowledge of a patients cultural background to provide the best possible health care. d. Understands and attends to the total context of the patients situation.

d. Understands and attends to the total context of the patients situation. Culturally competent implies that the caregiver understands and attends to the total context of the individuals situation. This competency includes awareness of immigration status, stress factors, other social factors, and cultural similarities and differences. It does not require the caregiver to speak the patients native language.

2. During a home visit, the nurse notices that an older adult woman is caring for her bedridden husband. The woman states that this is her duty, she does the best she can, and her children come to help when they are in town. Her husband is unable to care for himself, and she appears thin, weak, and exhausted. The nurse notices that several of his prescription medication bottles are empty. This situation is best described by the term: a. Physical abuse. b. Financial neglect. c. Psychological abuse. d. Unintentional physical neglect.

d. Unintentional physical neglect. Unintentional physical neglect may occur, despite good intentions, and is the failure of a family member or caregiver to provide basic goods or services. Physical abuse is defined as violent acts that result or could result in injury, pain, impairment, or disease. Financial neglect is defined as the failure to use the assets of the older person to provide services needed by him or her. Psychological abuse is defined as behaviors that result in mental anguish.

30. An older adult patient in a nursing home has been receiving tube feedings for several months. During an oral examination, the nurse notes that patients gums are swollen, ulcerated, and bleeding in some areas. The nurse suspects that the patient has what condition? a. Rickets b. Vitamin A deficiency c. Linoleic-acid deficiency d. Vitamin C deficiency

d. Vitamin C deficiency Vitamin C deficiency causes swollen, ulcerated, and bleeding gums, known as scorbutic gums. Rickets is a condition related to vitamin D and calcium deficiencies in infants and children. Linoleic-acid deficiency causes eczematous skin. Vitamin A deficiency causes Bitot spots and visual problems.

4. Which statement is best for the nurse to use when preparing to administer the Abuse Assessment Screen? a. We are required by law to ask these questions. b. We need to talk about whether you believe you have been abused. c. We are asking these questions because we suspect that you are being abused. d. We need to ask the following questions because domestic violence is so common in our society.

d. We need to ask the following questions because domestic violence is so common in our society. Such an introduction alerts the woman that questions about domestic violence are coming and ensures the woman that she is not being singled out for these questions.

20. A patient has had a cerebrovascular accident (stroke). He is trying very hard to communicate. He seems driven to speak and says, I buy obie get spirding and take my train. What is the best description of this patients problem? a. Global aphasia b. Brocas aphasia c. Echolalia d. Wernickes aphasia

d. Wernickes aphasia This type of communication illustrates Wernickes or receptive aphasia. The person can hear sounds and words but cannot relate them to previous experiences. Speech is fluent, effortless, and well articulated, but it has many paraphasias (word substitutions that are malformed or wrong) and neologisms (made-up words) and often lacks substantive words. Speech can be totally incomprehensible. Often, a great urge to speak is present. Repetition, reading, and writing also are impaired. Echolalia is an imitation or the repetition of another persons words or phrases. (See Table 5-4 for the definitions of the other disorders.)

30. When providing culturally competent care, nurses must incorporate cultural assessments into their health assessments. Which statement is most appropriate to use when initiating an assessment of cultural beliefs with an older American-Indian patient? a. Are you of the Christian faith? b. Do you want to see a medicine man? c. How often do you seek help from medical providers? d. What cultural or spiritual beliefs are important to you?

d. What cultural or spiritual beliefs are important to you? The nurse needs to assess the cultural beliefs and practices of the patient. American Indians may seek assistance from a medicine man or shaman, but the nurse should not assume this. An open-ended question regarding cultural and spiritual beliefs is best used initially when performing a cultural assessment.

9. When assessing the quality of a patients pain, the nurse should ask which question? a. When did the pain start? b. Is the pain a stabbing pain? c. Is it a sharp pain or dull pain? d. What does your pain feel like?

d. What does your pain feel like? To assess the quality of a persons pain, the patient is asked to describe the pain in his or her own words.

31. The nurse is preparing to complete a health assessment on a 16-year-old girl whose parents have brought her to the clinic. Which instruction would be appropriate for the parents before the interview begins? a. Please stay during the interview; you can answer for her if she does not know the answer. b. It would help to interview the three of you together. c. While I interview your daughter, will you please stay in the room and complete these family health history questionnaires? d. While I interview your daughter, will you step out to the waiting room and complete these family health history questionnaires?

d. While I interview your daughter, will you step out to the waiting room and complete these family health history questionnaires? The girl should be interviewed alone. The parents can wait outside and fill out the family health history questionnaires.

6. During an examination, the nurse notices a patterned injury on a patients back. Which of these would cause such an injury? a. Blunt force b. Friction abrasion c. Stabbing from a kitchen knife d. Whipping from an extension cord

d. Whipping from an extension cord A patterned injury is an injury caused by an object that leaves a distinct pattern on the skin or organ. The other actions do not cause a patterned injury.


Related study sets

Jensen's Health Assessment 3rd Ed. | Chapter 13

View Set

Intro to Sociology Chapter 1-7 Midterm Review

View Set

12 Chapter Quiz: Informative Speaking

View Set